Download as pdf
Download as pdf
You are on page 1of 103
MUC LUC 1 Cée phép doi hinh phang § 1 Dai cuong vé cdc phép bien hinh va céc phép dai hinh pring vee Whore § 2. Sw-xéc dinh va dang chinh téc cia mot phép di hinh phing +. phing, Thi du minh hoa va bai tap 1 Khai nigm vé phép bién hinh Dinh nghia phép bign hinh . ‘Thi du vé phép bign hinh see eee Bai tap vé phép bien hinh trong tap hgp diém T Cée phiin tir bat bign trong mot phép bign hinh. Phép bien hinh dionguoc ... +.» ‘ich ede phép bign hinh Vé sut xde dinh mot phép di hinh phang. .... « Quan he gitta cde phép tinh tién phép quay ph: ic phép d6i xting - truc trong mat phing . Dang chinh te cia m6t phép dai hinh phang . Phan loai cdc phép ddi hinh phang (Bai doc them bé GLVAGQ) eee eee eee phing § 3. Van dung phép ddi hinh vio viee gidi mot s6 bai ton hinh hoc ‘Ung dung phép tinh tign va vige khio hinh va dung hinh, «6. + ‘Ung dung phép doi xing tam vio kho sit tinh chat hinh va dymg hinh Nhiing digu cin luu § khi vige gidi toan hinh hoc Ung dung hep i xing = true vio vige gid toa hinh hoe phang . Ung dung cée phép quay va ddi hinh (ndi chung) gidi toan seen eee eee Cau héi va Bai 26 26 28 2. Cac phép déng dang phing 41 § 4 Surxdc dinh va dang chinh tic ctia mot phép déng dang pring 2 4 1 Dai cuong vé c: bhp dng dang ping.» + 4 2 Phép vite -. 43 3 Suexéc dinh mot phép déng dang ph - 48 a a 50 5 53 § 5. Van dung phép déng dang vio viee iia 36 bai toén hinh hoe phang - Thi du minh hoa a 55 1 Ung dung phép vi tu vao giai toan hinh hoc + 55 2 Ung dung phépdéng dang thuan (vi tu quay) vao viee gidi ton hinh hoc . . « = 60 3 Ung dung phép déng dang nghich (vi wr - di ximg) vao vigc gidi todn hinh hoe . : seeeees 65 4 Bai tap van dung phép déng dang vao vige giai t 67 3 n 71 -B 8 80 81 4 87 87 89 89 phang 91 § 5. Suxde dinh va dang chin ti § 6 Van dung phépdong ds phang ‘Tai ligu tham khio Chuong 1 Cac phép doi hinh phang § 1 Dai cuong vé cdc phép bién hinh va céc phép doi hinh phang 1 Ki Cho hai tip hop diém T- va-T" ta goi L& mot song Anh tir T va0 7", moi phép tuong ting / ma vi méi diém M cita T d&u duge gin v6i mot diém M’ duy nhat cia T", ky higu kM! = f(M). igm vé phép bien hinh Nhu vay, cho mot song 4nh f : T+ T! vio T' LA cho mot quy tie dé, v6i bat ky mot diém M € T bao gidsta ciing 6 mot dim f(.M) hon ton xéc dinh cia T' sao cho (i) Neu M va phan bigt hai diém phan biét cla T thi f(.M) va f(N) 1a hai diém aT! MAN thi f(M) 4 f(N). (ii) Voi VM! ET’ thi bao gid ciing c6 mét diém M € T sao cho f(M Diém M' = f(M) duge goi Ia anh, hay diém tong Ging hode hinh bigh cia diém M qya anh xq f. Nguge lai, diém M duge goi 1a tao anh ciia diém M'= f(M) qua anh xa f. Néu M! = f(M) thi ta con néi ring énh xq f @& day [a mot song Anh r6i) bien diém M cia T thinh diém M' ciia T" Khi hai tip hop diém T va T' 1a déng nhat, cing c6 nghia 1a tring nhau, ky higu T = 7", ta néi rang f 18 mot phép bign hinh trong T (hay tir T vio chinh 16). Nhut vay, ta 66 thé din nghia mot phép bign hinh trén dung thing, trong mat pha Anio dé trong mat phi P hay T [a tap hop tat digm ciia khOng gian K, Tham chi T c6 thé fa tap 1 iS digm ciia mot hinh H{ ndo dé 1 mot bo phan (Lap con) cla dudng 12 A, hay mot bo phan ciia mot mat phang P hay mot bd phan cla khong. ; ky hieuH CA, HCP hay H.€ K. Tacé dinh nghia sau dau inh nghia phép bién hinh 1A A’ hoae P + P tir tap cée diém cita duig thang A 1g P len chinh né duge goi a mot phép bign hinh trén dudng guy tic dé véi moi diém cita P ta tim duge mot diém M! = f(1M) hoan ton x4c dinh, tho’ man hai diéu kign sau day (i) Neu M va N déuthuge P, M 4 N thi f(M) va f(N) déuthudeP. f(M) A F(N) (ii) Véi VM! € P thi t6n tai duy nhat digém M €P sao cho f(M)=M'. ia P thi ta c6 thé xAc dinh duge tap hop diém {(M),M © 1}; f(74) fa mot hinh phing duoc goi la anh hay nh tuong ing cia hinh H qua phép bién hinh f nguoe Ia go dinh (hay hinh nguyén Néu H 1a mot hinh no dé H = f(H) hinh bién déi ho hinh 1 dugc goi f Hinh 1, iia hinh (#4) qua phép bien hinh Hinh 1 Chi thich. Phép bién hinh dinh nghia nhu trén con duge goi mét cach chinh xc hon 1A phép bign hinh diém (vi né bign déi diém thinh diém). Hai phép bién hinh digm / va f’ 1a twong duong néu véi moi diém Mf cla T déu c6 cing mot nh trong T: VM T suy ra M' = f(M) = f"(M)p ta viet f" =f. 3 Thi du vé phép bién hinh ‘Thi dy 1. Trong mat phang cho mot dutmg thang A va mot diém O cé dinh tren A. Véi méi diém MI ZO tren A ta lay diém M' doi xting vi M qua O: digu d6 cing c6 nghia la MM’ duoc hoan toan xac dinh bai dang thiic vée ta OM'=-OM (ay Néu M tring O thi ta ldy M' =O. Thé thi r6 rang 1a dmh xa fs Mo M! tir A — A xée dinh nhur tren 1 mot song Anh tir duimg thing A vao chinh né, hay theo dinh nghia thi f 14 mot phép bign hinh cia duimg thing A. Dé chinh 1a phép doi xing tam O cia dung thing A, ta ky higu la ‘Do hay ‘D(O). Vay DO): Mis MitA > AvAT =A. Chi thich. Trong tnaing hop nay ta cing e6 D(O) : M++ M. Neu T =P va M! = f(M) duge xéc dinh béi (1) thi f > M+ M! tir P > P la mot phép d6i xing tam O cia mat phing P. Thi dy 2. Goi A,B, vaC, lin luot Ia wung digm céc eanh BC,CA va AB mot tam giée ABC cho truéc trong mat phing, Véi mé M cia mat pl ta lay Lin lugt céc diém A’, B', va C! doi xiing voi M theo thiity qua 4,, Bo. Co. Chitng 16 ring iém M' ndo d6; 1/ Céc dung thing Ad’, BB! va CC! déng quy 6 mot 2/ Anh xa f : M ++ M" tir P vao chinh né 1 mot phép bign hinh cia mat phang. Hinh2 ‘That vay, tir gid thiet dé ding suy radiém M 1a dinh thé wr chung cia ba hinh, binh hinh BA’CM,CB'AM, va AC'BM. Tit d6 suy ra BCBIC'.CAC'A' va ABA'B! déu la nhiing hinh binh hinh va mdi hinh ny nhan hai trong ba doan thang AA’, BB! va CC' lam ce dutmg chéo. (Hink 2). Do di ba dutmg thing A’, BB! va CC! doi mot cit nhau 6 ciing mo 46 1a trung diém chung A" ciia ca ba doan thang 46. Nhu vay i cia mat phing P 4a chi ra mot quy tic f x4c dinh hoan toin diém f(M) = M" nhu di cho 6 tén va do dé f 14 mot dan dnh. Dio lai, véi mdi diém AV" ciia mat phang ta Idy cdc diém A’, B,C! theo thit tu déi xing voi A,B,C qua M'. The thi r6 rang MI" 1a tam chung cia ba hinh binh hanh BCB'C',CAC'A! va ABA'B', Bay gids ta lly diém M d6i xing véi A! qua Ao: néi khée di, M 1a dinh thi t cia hinh binh hinh BA'CM. Tir dé suy ra M [a dinh thé we chung cita cd ba hinh binh hinh BA'CM,CB'AM va AC'BM. Bai vay, M d6i ximg v6i B' qua B,, déng thai cling doi ximg voi C! qua C,,. N6i khéc di la, A.A’, B,B" vaC,C" dong quy 6 diém M. iém M' bat ky cilia mat phang P ta dai chi ra duge cach M". Vado d6, f Nhu vay 1a, vim xée dinh diém M va diém M nay 1A duy nhat sao cho f(M1) 1A mét fon anh. Kétluan, Anh xa fs M+ M! tit P +P vita 1a don én, vita 1a todn anh nén theo dinh nghia, /' 14 mot phép bign hin cilia mat phing. Chi thich. Néu goi G 1a trong tam ciia tam giée ABC , ta cdn ching minh duge ring dudng thing MM" dia qua G va cip dim M,M' thoa man he thie vécto OM! 2) He thie (2) ciing néi len rang quy tée f > M++ M' tir P — P chi ra trong thi dy 2 tuong ducmg véi quy te (nh xa) f", trong dé véi mdi diém M bat ky cia P ta xéc dinh duge duy nhat diém M"€ P theo (2), Ta sé tré lai thi du 2 nay 6 phain sau, § 4. 4. Bai tap vé phép bién hinh trong tap hop diém T 1.1 Trong mat phing cho mot dutmg tron (O) va hai tigp uyén a va b cla n6. Mot tigp tuyén t thay di céia (O) cat a va b theo thaw M va N, Héi énh xa f : Mv N tira dén b cé phai 4 mot song 4nh khong? Xét hai dudng trutng hgp a song song véi b va.a cat b. 1.2 Trong mat phang cho hai dutmg thing phan bigt a va b. Goi A va B theo thd tu 1A cdc hinh chiéu trén a va b ciia mot diém M bat ky ctia mot mat phing; A" 1a diém d6i xing v6i M qua tung diém 7 ciia doan thing AB. Ching minh ring anh xa f : MH» M' tir —+ P 1a mot phép bién hinh cla ‘mat phang khi va chi khi a vad khong vuong gée v6i nhau. 1.3 Trong mat phang cho hai duéng thang a va A cat nhau (nhung khong vudng g6c) & O va mot diém P c6 dinh khéc O sao cho O 12 hinh chiéu clia P ‘Tie day tr di, ndu Khong c6 chi thich gi them, ta higu hinh chigu 1A hinh chigu wuong gée. tren a. Véi m r6i duimg tron ta digm M ten A, ta dung duimg tron (OPM) cit lai aC, mC diqua M cat lai A&.N a) Chimg minh ring énh xa f : M+ N tir A vao chinh né 14 mot phép bign hinh cita duémg thing A. b) Hay ching minh ATV khong déi v6i moi M tren A. Tir dé c6 thé goi ten duige phép bign hinh f ciia A. 1.4 Gi sit 1 1a mot dim bat ky |BC dicho. Dung cdc duimg thing Ax, By vi Cz tuong ting doi xing duong thang AM, BM, va CM lin luot qua phan gidc ciia cdc g6e A,B va C cia tam gide ABC. a) Néu M quy & mot b) Ching minh ring énh x phép bien hinh cia mat phing. 1+ M' tir P > P khong phai fa mot ©) Chimg minh ring quy tich nhiing diém M trong mat phing sao cho Ac || By || Cz la dutmg tron ngoai tigp tam BC d) Tim tap hgp diém T (T c P) dé anh xa fs M > M' tit T > Ta ‘mot phép bién hinh cila tap hop diém T, mot bd phan cita mat pl P. 5 Cac phan tir bat bién trong mot phép bién hinh. Mot diém M nao dé cita tap diém T ma tring véi anh f(M) cla n6 tong mot phép bign hinh diém f cia T goi 1 mot digm bat dong hay diém kép doi vGiphép bién hinh f wong T: M eT ma f(M) =M, M 1a mot diém bat dong a f trong T. Phép bién hinh cia mat phang ma moi diém cia mat phang déu m bat dong goi la phép bién hinh déng nhat hay goi tat 1a phép déng nhat, fa Td hay e, f = Idkhi va chi khi M = j(M), VM EP. I ky hi Moi hinh H (cla P) ma ting véi anh (hinh bign déi) H! = f(72) celia né trong phép bién hinh f duge goi 1A mot hinh bat bign d6i véi phép bien hinh f cia P. Néumot hinh 2 bat bien ddi v6i f trong P mA moi diém ciia né déu bat dong thi H. duge goi la mot hinh c6 dinh doi véi f. Ching han, trong phép d6i xing ‘D(O) thi tam d6i xing O 1a mot diém bat dong duy nhat va moi dutng thing qua O déu Ia bat bign. Trong phép doi xing D(A) truc A thi true déi xing A 1A hinh c6 dinh cdn moi dudng thing vuong 6 g6c vai A déu IA bat bién. ‘Trong phép tinh tign T(%) theo vécto 7 khde vécto khong thi khong ¢6 diém bat dong ndo, nhung moi dudng thang c6 phuong cila 7 (tie song song véi 7) déu 1a bat bién, 6 Phép bién hinh dao nguoe Dé thay ring moi song Anh f tit J —> T" déu xde dinh mot song anh f" khéc tir 7! T, goi la Anh xa nguge cia f va ky higu la f~ f-. Khi 7! thi f~! goi lA phép bien hinh dio ngugc cia f tong T- Dé thay ring, phép tinh tién T(@) theo vécto 7 khde vécto khong 6 phép bign hinh dao ngugc ciing Fa mot phép tinh tign, nhumg theo vécta —} con phép d6i xting tam D(O) hay phép doi xting true D(A) thi phép dio nguye tring véi chinh né: D-(O) =D(O),D-(A) = D(A) Mot phép bign hinh f celia P duge M €P tring véi anh cia digm tuong tng M! f(M) thi M= f(M"). i 14 phép bién hinh doi hgp nu moi diém {(M) ciané: VME P: M'= Nhu vay, / 18 phép bign binh doi hgp khi va chi khi f= f-! Phép doi xing tam va phép doi xing tim va phép doi xtng truc fa nhimg thi du vé phép bién hinh doi hgp trong mat phing. 7 Tich cdc phép bién hinh Gia sit fi va fo 1a hai phép bien hinh cia mat phing f, : P + P va Ja: P + P; M la mot diém bat ky cla P. The thi, VM P, IM, € P va M! €P sao cho My = f(M) va M! = fa( Mj) vadods, anh xq f : MM! tir P — P cing 1a mot song anh, nghia la f true tigp bién M thanh M1 ciing 1a ‘mot phép bign hinh ciia P. Nhu vay, tit hai phép bign hinh fy va fy ciia P ta da xée dinh mot phép bién hinh f cing cia P goi la tich cla hai phép bien hinh f; va fo, ky higu Ia foo fi. Vay tacé M!= f(M) = falfi( M), VM khi va chi khi M' = foo f(M) vado d6 f = fao fi. Cling vay, tich fo foo fi = fao (fre fa) Ia tich cf phép bign hinh (fy0 f,) va fs theo thitty dé; tich fo fz0 fae fy Ia tich iia phép bign hinh (fo foo fi) va fa theo thi ty 46, v... iin luu ¥ rang phép bign hinh f = fro fy 1a két quad cita vige thyc hién lién tigp hai phép bien hinh: phép bién hinh thi nhat Ia fy va phép bien hinh th hai 1 fo, con g = fro fa cling fa mot phép bign hinh nhung duc thuc hign theo thit turnguoc lai, Noi chung, tich f = fyo fy vatich g = fio fa li hai phép bien inh khée nhau. ‘Tich cde phép bign hinh c6 nhimg tinh chat sau day 1/ Két hop, nghia la fyo(fro fi) =(fsofe)ofi = fae foo fy. Nhu vay, bao gid cling c6 thé thay hai hode nhiéu phép bien hinh lien tigp bai tich ciia chiing, hoae nguge lai, ¢6 thé thay mot phép bién hinh no dé bi mot tich tuong dutong. 2/ Néi chung, tich hinh fy va fo goi ic phép bien hinh khong giao hodn; tich hai phép bién giao hoin néu f,o fo = foo fy. 3/ Trong tap hop cdc phép bign hinh cia digm P, phép déng nhat Jd 1a phan tirtrung hod trong cde phép tofn tich Vf: Ido f=fold=f 4/ ‘Tich hai phép bign hinh dito ngugc nhau 18 phép déng nhat faof=fof'=ld, (vf) 2 = Id khi vachi khi f = f~! khi va chi khi f 1a ra, phép bign hinh dionguge cia tich f2o fy Tir dé suy ra fof phép bién hinh doi hop. Ngo’ Iatich fyto fy, vay (hofiy 1g sot off ‘That vay ta 06 (fro filo(fi of) = ho (fief) ° fs" = froldo fy! fro(Ido fz = frofy' Id 8 Dai cuong vé cae phép doi hinh phang a) Dinh nghia phép ddi hinh ky higu 18D néu bat ett hai F(N) eta ching ta déu doi hinh ciia mat phang, hay g bio ton khoang céch gitta bat cif hai digm mio ciia mat phang. Vay 12 Ff €{D}citaP khi va chi khi f(M)f(V) = MN, (VMN EP) {D} 1a ky hiéu tap cdc phép dai hinh cia mat phing. Chi thich. Chinh vi phép ddi hinh bio ton khodng each gitta bat ett hai digm nao nén ngudi ta goi n6 Ia phép bigh hinh ding cy hay goi vin tit la phép daingew. Tir dinh nghia ciia phép ddi hinh ta suy ra cdc hé qui. a. Phép bigh hinh déng nat Jd 1A mot phép di hinh b. Phép bien hinh dio nguge cha mot phép doi hinh cling L& mot phép ddi hinh. c. Tich cia hai hay nhiéu phép ddi hinh 1 mot phép dai hinh. d. Phép tinh tién, phép doi xiing tam, phép doi xing truc 1a nhimng phép ddi hinh, b) Cae tinh chat cata phép déi hinh Dinh ly 1. Phép doi hinh bio toan su thing hing cia ba diém va thif ty ciia chiing trén duémg thang chifa ba diém va the ty cla ching trén dung thang chua ba diém 46. Cu thé Fi Phép ddi hinh bién ba diém thang hing A, B,C thinh ba diém A’, B’,C' thing hang ciing theo thét wr d6, Phép chimg minh xem nhut 1a bai tip cho ban doc. Hé qua 1. Phép doi hinh bién mot dutg thang thanh mét dudng thang, bién mot tia thinh mot tia, bién mot doan thang thinh mot doan thang nao 6. Hé qua 2. Phép ddi hinh bién mot tam gide thanh mot tam giée bing né, bien mot gée thanh mot géc bang n6, bign mot dudng tron thanh mot dudng tron bing né, trong 46 tam bien thinh tam. Dinh ly 2. Mot phép dai hinh phang cé ba diém bat dong khong thing hang 14 phép bign hinh déng nhat. Chitng minh. Gid sit f : P + P 1a mot phép ddsi hinh phing c6 ba di bat dong khong thing hing (Hinh 3): A= A! = f(A),B = B! = f(B) va C=C" = f(C). The thi, theo tinh chat cia phép dds hinh, bat cit mot diém nao trén cde dutmg thing (BC), (CA) hoic (AB) déu 1a diém bat dong (hay ching minh digu d6), Tir d6 dé ding suy ra moi diém M cia mat phang (ABC) déu la digm bat dong, va do 46 f = Id Hinh 3 H@ qua 3. Mot phép dai hinh phing D 4 Id thi hoie khong ¢6 diém bat dong nio hoje c6 mot diém bat dong duy nhat, hoe c6 mot duémg thing mA moi digm cita né déu 1a digm bat dong, tiie 12 c6 mot dutmg thing c6 dinh. 9 Bai 1.5 Chiing minh rang phép doi hinh bao toan su thang hang cita bat ky ba diém nao, do d6, bién dutng thang thinh dutg thang (Dinh ly 1). vé dai cuong c4c phép doi hinh phang 1.6 Ching minh ring phép doi hinh bio to’n quan hé song song cita hai dutmg thang va Khong cach gitta ching, nghia la, néu a || b,a’ = D(a), = D(b) thi a’ || b’ va d(a’,¥’) = d(a,b), trong dé d(x,y) chi khoang céch gitfa hai dung thang song song x va y 1.7 Hay sit dung phuong php ching minh phan chimg, néu trong Dinh ly 2 ciia phép ddi hinh phing. ‘hing minh tinh chat 1.8 Ching minh chi tiét hon He qua 2 cia Dinh ly 1 vé tinh chat ciia D, tite 1a Phép doi hinh bién mot tam gidc thinh mot tam gide bing n6, trong dé trong tam G, try tm H, cic tam T va O cia cdc dutmg tron noi tigp, ngoai tigp cla tam gide tg0 inh ABC theo thé ty bin thinh cc tam tuong ting GH!’ va O! chia tam gide anh A'BIC! 1.9 Ching minh ring trong mot phép tinh tien, anh d’ ella mot duimg t hoe song song véi d hode tring d. Tir dé suy ra tip hgp tat ed ede dutng thing bat bign trong mot phép tinh tign. 10 =a thinh 1.10 Ching minh ring phép d6i xting tm D(O) bign mot véeto MI} mot vécto déi MN véiné, tie Ba MN iD a. Tir dé suy ra ring phép d6i xting - tam bao ton phuong cita moi dudng thang; déng thoi tim duge tap hop tat ca nhing dutmg thang bat bign qua phép d6i xting - tam. 1.11 Ching minh rang phép d6i xting - tryc D(A) bién mot dutmg thing a thinh dudmg thang a’ d6i xing véi a qua tryc A. Tirdé tim duge nhing phyong dutng thing bat bién trong mét phép déi xing - truc. 1.12 Hay tim nhimg phép dai hinh D bign mot tam giée déu ABC thinh chinh n6, 1,13 Tim nhiing phép doi hinh D bién mot hinh vuong ABCD thanh chinh nd. 1.14 Tim nhing phép doi hinh D bign mot hinh binh hanh ABCD thanh chinh nd. 1.15 Tim nhiing phép dai hinh D bign mot hinh chirnhat ABCD thinh chinh né. 1.16 Chimg minh ring tich ciia hai phép d6i xting tryc ¢6 cde true vuong géc & digm O 1a phép d6i xing tam D(O). Nguge lai, mot phép doi ximg tam ‘D(O) c6 thé phan tich bang vo s6 cach thinh tich ciia hai phép doi xing. true D(A;) va D(A») mign 12 hai tryc Ay va Ao vudng géc véi nhau 6 O. § 2 Su xde dinh va dang chinh tac cia mot phép doi hinh phang Khi ching tandi cho bign hinh f cia ‘P (mat phang) tie Ia chi ra diy di ton phép dai hinh (hay phép bign Ot phép ddi hinh D ma téng quat hom 1a cho mot phép fc you 16 dé xéc dinh hoin V6i mdi diém MI bat ky cia P ta phai chi ra cach xée dinh duge diém tuong img (anh) M' ciia né qua phép dd bign hin) nay. dung, ciing Ia hinh (hay phép Vé phép dds hinh, ta da biét ring mot phép doi hinh bign mot tam gi thinh mot tam gic A’B'C" bing n6, trong dé cc canh tuong Ging bang nhau, g6c tuong Ging bing nhau. Ménh dé sau day khang dinh digu nguge lai BC ic 1 Vésu xéc dinh mét phép doi hinh phang Dinh nghia 3. Cho ABC va A’B'C' 1a hai tam fing nhau cho truée trong mat phang P véi B'C' = BC,C'A! =CA,A’B! = AB. Bao gid ta cing 6 mot va chi mot phép dai hinh D : P+ P bign A thanh A’, B thinh B! va ul thinh C', Déng thoi, phép di hinh D nay c6 thé phan tich thanh tich ca khong qua ba phép doi xiing true. Chiing minh. Trude hét, dé trinh bay duge gon ging ta quy uée ky higu nhur sau: ¢[MIN] 18 trung tryc e ‘N; ‘DA, ode ‘D(A;) chi phép d6i xiing true c6 tryc 1a A;, trong 46 7 1a chi s6 1,2,3. Hinh4 Tén tai. Néu Ava A! 1a hai diém phan biet, ta goi Ay = ¢[AA’]. Qua phép ‘D(Ay) tam giée ABC bien thinh tam giée A’B,C; bing né nén AA'B,C, = AA'BIC', do dé A’B, = A'B', (Hinh 4.) Néu By Bl, ta goi Ax = ¢[B,B'] thi theo chimg minh ten, A! € A. Bai vay, qua phép D(A2) tam gide A'B,C; bién thinh tam giée A’B'C, big 6, nén AA'BIC) = AA'B'C! va do dé A'Cy = A'C!, BICy = BIC'. Dén day, néu Cy ZC’, ta goi Ay = ¢[C,C"] thi A! va B! déu thuge As. Va eudi cing phép doi xting truc D(Ag) bign tam gide A’B'C thinh tam gide A’B/C’. Nhu vay Ia, thyc hign lign ti¢p ba phép ddi xing tryc D(A) voi i = 1, f ='D(Ag)oD(Ay)o'D(Aj) 1a mot phép doi hinh D bién tam gide / tam giée A'B'C' wong 46 A! = D(A), B' = D(B) vac! =D(C). ‘Trong qué trinh ching minh, ching ta da ba lin sit dung dén ti\néu" (cong 6 nghia 1a \gid sirring”) vA m6i Kin nhu thé déu xust hign mot phép d6i xing rye D(A,) theo thé ty i= 1,2,3 Vi nhan xét nay, ta thay ring néu A! = A thi khong phai thyc hign ‘D(A,). Néu By = BY thi khong phai sir dung dén (As) va néu C2 = C’ thi ciing khong phai sit dung dén D(Ag). 6 mot cap diém trong ba cap diém chi ra 6 tren sO phép d6i xtig truc c’in phai thyc hign gidm di mot. N6i t6m lai, mé tring nhau thi Duy nhét. Gia sit rang c6 hai phép ddi hinh D, : P > P va D, 2 P > P 12 cing bign tam gidéc ABC thinh tam gide A'B'C’. Thé thi phép bién hinh (ddi hinh) tich Dz1D, ciing 1A mot phép ddi hinh D bign A thanh A, B thinh B va C thinh C. Nhu vay 1a phép di hinh D = D; 10D, c6 ba diém bat dong khong thang hing v nhat (Dy'oD, Td) va do dé Dy 0(Dz!0D,) =D20ld=Dz a Mat khéc, nhi tinh chat két hop cita tich cdc phép big hinh ta lai cd Dz 0(Dz!0D) = (DsD;!)oD, =IdoD, =D, Q) Di chiéu (1) va (2) ta suy ra Dy =D}. Va Dinh ly 3 da duge ching minh. HG qua 1. Tich cia n (1 0, cho truéc (0 < a 4 277) Ngodi g6e dinh huéng cita hai vécto duge sitdung dén trong phép quay ngudi ta cling cdin dén va sirdung g6e dinh huéng cia hai dudmg thang. Chang han, néu phép quay Q(O,a) tim O géc a (mod 277) bigh vécto AB thinh vécto A’B', thi (AB, A'B') =a mod 27. Néu xem AB va A'B", Ia hai truc thi géc dinh huéng, 2 gitta hai rue AB va A’B' cling bing g6c gidta hai vécto d6, va xde dinh sai khde 2kz. Nhung phép quay Q(O,a) da bién AB thinh A’B' thi cing bign dutmg thing (AB) thanh dutmg thang (4'B") sao cho (AB, A'B') =0-+2kr, va ((AB),(A'B)) a+ kr, KEZ 18 trong lie géc thong thudng cia hai duimg thing AB va A’B', theo dinh nghia, thi bing a hode t =a néu0 C. Dung diém Q = ‘Da(P), ta due CP =CQ ZQCB = ZPCA = 20°. Tir dé suy ra tam gid CPQ 1a déu (Hinh 20) va tinh duge ZBQP = 150°. Vi vay ZBQP = ZBQ BQP Bai thé, BQ khong nhiing 1a cla gée ZCBP ma cdn li phan a gée ZCQP dong thdi phép d6i xing truc D(BQ) bign ABQC ABQP, tong d6 BC + BP, BP + BC va ZBCP + ZBPC. Viv ZBPC = ZBCP =20° +60" =80" va do dé tam giée BCP can 6 B c6 gc 6 dinh B bang 20°. i lai: ZBPC =80° ‘Thi du 7. Trong mat phang cho hai dutng thang d; va dy cat nhau & O va ém P c6 dinh nim ngoaid;, 2. Mot dumg thang A quay xung quanh P dutmg thing A, Ao d6i xing vi A Lin lugt qua di; va dy cit nhau & M. Tim hay quy tich elia M. Nhén xét. Ching ta nhan thay c&e you t6 di xting = truc da xuat hign ngay trong cdc dit kign cla bai todn, vi vay bai todn nay ddi hoi phai sit dung tinh chat cia phép d6i xting - truc va géc dinh huéng cita hai dudng thing (mod m) dé tim quy tich. Hinh21 ém d6i ating vi P quad, i= 1,2 (Hinh 21) Vi P € A nen suy Goi Pla ra Ped, Da, (A). tir dé ta ta duge (AnA)=2(Arsd:) =2(disA), (mod 7) (A, Av) = 2(dz, Av) =2(A,d2), (mod 7) Tir d6 suy ra (AAs) =2(disdy)=25=6, (mod 7) a wong d6 (di,d;) =6, (mod m). Vi A: NAg = M va Pre A, (1= 1,2) nen (1) duge viét lai nhu sau (MP,,MP,) =2(diyd2), (mod 7) “ Mat khéc, vi P; = 'D,,(P), (i= 1,2) va O = di Nd nén ta c6 cde ding thite (OP,,OP) =%(A1,d;)=2(ch,0P), (mod 7) 2) (OP,OP2) = 2(d,0P,) = 2(OP,d2), (mod m) ) Tir (2) va (3), nhd he thiic Sald ta duge (OP,,OP2) =2(didz), (mod) (ii) Di chiéu (i) va (ii) ta suy ra (MPjMP,) =(OP,,OP,), (mod m) (iii) Diing thite (iii) ching td bon diém O, Py, Py va M= Ay Ag cing thude mot duéng tron véi vi tri clia dudmg thang A quay quanh diém P c6 dinh. Vay, ta di én két luan a) Néu d; va dy khong vudng géc véi nhau 6 O va do d6, O, P; va P, khong thing hang thi {= A, A Ao} 1a duting tron ngoai tigp tam giac OP, P) b) Néu dy wudng géc véi dy thi {M} 1A duimg thang di qua P, va P: ‘Thi dy 8. Tim dutmg di cia mot qua bi-a, sao cho sau khi cham hai lin vao thinh ban, né di tir diém A dén diém B ( Hinh 22). Hinh 22 ‘Trude het, dé ¥ rng do téc dong cita lve day biin vio qua bi-a cha ngudi choi, qu bi-a sau khi cham vao thinh ban (ban bi-a c6 dang hinh chit nhat hoae vudng PQRS) thi phan xq tré lai theo quy lugt nhu phan xq Anh sing, te 18 g6e phin xq bing g6c ti tia phan xq d6i xing v6i tia t6i qua phap tuyCn cia thinh ban tai digm cham (Hinh 22), Nhu vay, 10 rang phép d6i xting true da phat huy tic dung giip ta gidi bai toan dung hinh nay, Goi AM va.N lin luot Ia dim cham Kin thé nha va Kin thi hai cita qua bi cécthinh ban RS va SP sau khi xusit phat tirdiém A (gn diém géc cia ban) tren matban dé réi tr lai diém B (gan diém gée P cita bin) cling da duge dinh sin tren mat ban bi-a hinh chit nhgt PQRS (Hinh 22). The thi, theo quy luat phan x9 n6i trén, dutmg thang MN chifatiaphan xa M va cing [a tia ti cita NV’ phai di qua céc digm A’ = D(A) va B' = D(B). Tir dé suy ra cach xéc dinh cfc vi tri M va N cita qua bi-a chamvaocéc thanh ban RS va SP : [4’B"/N{[RS],[SP]} = {MN} Két ludn, Dudng gap khic bon dot AMN'B 1a dung di phai cia qua bi-a sau khi cham hai lin lign ti¢p vao 4c thanh bin [RS] va [SP]. Chi ¥. Bién luan vé kha ning e6 Idi gi cde diém A va B (da dat san trén mat ban bi i ciia bai todn phy thude vao vi tri cla ). 5 Ung dung giai toan ¢ phép quay va doi hinh (néi chung) vao viée Thi dy 9. Trong mat phing cho hai tam gide ABC, ADE c6 cfc géc 6 dinh chung A bit nhau, déng thai AB 1 AD,AB = AD;AC | AE,AC = AB va hai tam gide 46 khong c6 diém chung nao khéc ngoai dinh A. Chimg minh ring duong thang chifa trung tuyén phat xudt tir dinh chung A ciia tam gisie nay cling chifa duimg cao ha tir A ciia tam gic kia. Chitng minh. That vay, theo gid thigt thi AB L AD va AC L AB. Digu dé 201 ¥ chiing ta lién tung t6i phép quay géc vung Xung quanh tim A. Hinh 23 Goi M 1a trung diém canh BC, H la chan dutmg cao AH cita tam gide ADE; ta ching minh AM DE. Mudn vay, ta thyc hign phép quay Q(A,7/2) (Hinh 23), Khi 46, néu tam giée ABC c6 hung thudn thi phép quay nay bién tam giéc ADE thinh tam giée ABF do D4 Bvi Es F = D(C). Vi BF =Q(DB), tong 46 Q = Q(A, x/2) nén (theo tinh chat ciia phép quay g6c x/2)thi BF L DE va BF = DE, dong thoi BF || AM va BF = 2AM (do M 1a dutmg wung binh cla ABC), Ti d6 suy ra AM 1 DE, cing tite 1a (AM) = (AH). Day 18 digu phai chimg minh. ‘Thi dy 10. (Ching minh dinh ly Pompiu) Néu P 18 mot diém bat ky nim trong mat phang cia mot tam gic déu ABC cho trudc thi bao gid ciing t6n tai mot tam giéc T c6 dQ dai ba canh bang céc khoding céch tir diém P dén c: tam gisic d&u di cho dé, ké ca truéng hop khi T suy bign thanh doan thing, ky higu7 = T(PA, PB. PC). Ngoai ra, hay tim qu tich clia diém P dé T suy bign. Nhén xét. Dinh ly Pompiu trén day cho ta biét mot tinh chat dic wrung cla tam giéc déu. Cé nhiéu cach chimg minh dinh ly Pompiu. Sau dy 1a each ching minh nhé sit dung phép quay géc 60°. Phéntich. Gidsit PA =max{PA, PB, PC}. Néu3T(PA,PB,PC) thi at phai tn tai mot diém Q trong mat phang saocho PQ= PB vaQA= PC (Hinh 24), Thé thi, hai tam giéc BAQ va BCP (da c6 BA = BC va AQ = CP) sé bing nhau khi va chi khi BQ = BP va do 46 khi va chi khi ZPBQ = 60°. Sr phan tich ndy lam nay sinh ¥ tuéng sit dung phép quay g6c 60° xung quanh diém B mot cach hat sic ty nbién, Hinh24 ‘That vay, néu tam gidée déu ABC da cho ¢6 huéng thuan thi, phép quay Q(B, +60") gitr bat dong dim B, bién C thinh A va bign P thinh diém Q (sao cho BPQ 1 mot tam gide déu vA cling c6 hung thujn) va do dé, bién tam BCP thanh tam gic BAQ (Hinh 24), Tirdé suy ra PQ= QB = PB 36 Néi khée di 18, QAP chinh 1a mot tam giée T(PA,PB, PC) ddi ién phép quay (Q(B, 60°). Day 1a diéu phai ching minh. Tam giée T nay duge sinh ra béi diém P tong ting sé suy bien thanh doan thang khi va chi khi P,Q, A thang hang, nghia la T(PA, PB, PC) suy bién tong, duong véi (QP, QA) = 0, (mod 180°) (1). Lai do tam giée BAQ bing tam gidc BCP nén ta c6 (QB,QA) = (PB,PC) hay l& (QB,QP) + (QP,QA) (PB,PC), (mod 180°); VP € mp(ABC) (2). Mat khic, do ABPQ la déu va cing huéng véi ABCA déu, nén ta c6 (QB,QP) = (AB,AC) = 60°. (mod 360°) (3). Déi chiéu (1), (2) va (3) ta suy ra T(PA, PB, Pe AQAP suy bign khi va chi khi (PB, PC) = (AB, AC) (mod 180°) (4). He thite (4) cho ta biét {P} dé T(PA, PB, PC) suy bién 1 duéng tron (ABC) Thi dy 11. (Gng dung tich ciia hai phép quay) Dung ra phia ngoai tam giée ABC (gid sit c6 huéng thudn) hai hinh vudng ACIM va BCNP. Ching minh rang néu gitt hai dinh A,B cé dinh va cho C khép nira mat phing mé (nia ditong) c6 bs Ia dung thang (AB) thi dudng thing MP luon di qua mot diém c6 dinh. Hay xc dinh digm dé, Hinh 25 Loi gidi. That vay, khi gid A va B 6 dinh, C chuyén dong sé kéo theo sit chuyén dong cia hai diém AZ va P. Va nguige lai, diém P chuyén dong kéo theo C chuyén dong, 161M chuyén dong nhd thye hign lién tip hai phép quay cing g6c lugt xung quanh cdc diém B va A. Dig dé néi len ring M duge suy ra tit P boi phép ddi hinh D 1A tich ciia hai phép quay Q(Bz/2) va Q(A, 7/2) theo thé ty d6. Vay D 1a mot phép doi xing - tam, bign P thinh M. Dé xc dinh tam O cia D, ta hay phan tich mi phép quay Q(B, 7/2) ) thanh tich eiia hai phép d6i xting ~ true dé dua D vé dang chinh tic ‘ap 2,8), ta duge dang phan tich sau D=[D(Ar) oD(AB)] o[D(BA)o D(BY)] = D(Ax) 0 D(BY) By Wi hai tue doi xing duge xée dinh bai: (By, BA) = trong dé Ar 37 7/4, (mod) 2). Goi O = AvM By, ta duge OAB Ia mot 1g can & O va c6 hung thuan (Hinh 25). Vay O dinh thude nita mat phing mé (duong) chifa C’ vac6 bd 1a dudng thing AB, hodn ton duge xée dinh béi cdc ding thie (2), ‘D(Av) oD(By) = D(O) bién P thanh M va do 46, doan thang M/P luon nhan diém O c6 dinh lam tung diém véi moi vi tricia C thude i 1g (mo) dug c6 bw 1a (AB). Thi dw 12. Trong mat phing cho mot cung tron AaB. Mottia Av quay xung quanh A, déng thoi mot dong tir M ciing chuyén dong trén tia d6 sao hé Ax cat cung AaB émot diém N nao dé thi luén luon ta c6 AM = BN. Tim inh vach nén bai M khi V vach nén cung tron da cho. ythi {N} (BN, AM) =—g (mod 2r), ing thife nay hoan todn xéc dinh mot phép doi hinh D (xem Hinh 26 Ava y £0, (mod 27) nénD Ia Dé thay rang D cé diém bat dong (1am quay) duy nhat 1a trung d AaB (6 dé M J) va do d6 D Ia phép quay Q(w,— =P, (mod 27). Phép quay nay bién B think A va A thinh A’, Dé § ring khi thi M hép quay. cia cung g6c quay dinh béi he D(B) doi xing DAwA!) = —p vawA! = wA. N6 cdn cho ta biét A’ v6i B qua (Aw) (Hinh 26). Két lun, Quy tich cia M 1a cung tron AoA’ duge suy ra tir {N} = AaB bai phép doi acing truc D(Aw). 6 Cau hoi va Bai tap A. Vin dung phép tinh tign vao vie giai ec bai toan sau day 38 3.1 Chimg minh ring t6n tai mot tam gic ABC nao dé va dign tich ca tam gide T ma cée canh bing céc dutmg trung tuyén cla mot tam giée ABC no dé va dien tich cia T bing 3/4 dign tich tam gide ABC. 3.2 Hinh binh hanh ABCD c6 dutmg chéo AC = a, Qua A ké cae dudng cao AR va AF xung cdc canh BC va CD. Tinh khoang cdch tir A dén true tam H cia tam giée AEF, cho bittg EF =b(b< a). 3.3 Trong mat phing cho mot dudng thang A va mét diém A cé dinh. Mot dung tron (v) c6 ban kinhr cho trude chuyén dong trong mat phang nhung luon di qua A. Tin quy tich cdc tip diém ela cac giao tigp eiia (v) c6 phuong cia dutmg thing A da cho. chuyén, dong wren duing thing chia canh AB cla mot ABC sao cho MN = AB. Goi D va E Vin luot 1a binh chiéu cia M wen (BC) va cia N wen (CA); § la trung diém ciia AN va Q 1a tam dudng tron (CDE), Ching minh SQ 6 d6 dai khong di; tir d6 suy ra quy tich ciia Q khi vécto MV trugt trén dudng thing AB. 3.5 Cho dudng trdn (0) duimg kinh C'D va mot day cung AB khong cit CD. Tim trén dutmg tron mot diém P sao cho géc noi tiép APB chan trén dutng kinh CD mot doan thing MN c6 do dai bing ¢ cho truée. Bién lu: 3.6 Cho hai dung tron ngoai nhau (O) va (O') va mot dutmg thang d. Hay dung mot dubng thang c6 phuong ciia d eat (J) va (O') sao cho tong do dai cic day cung cia chiing dinh boi dutng thang c6 mét d6 dai £ cho tude. Bign luan. B. Van dung phép déi xiing tim vao vige gidi ec bai toan sau diy 3.7 Gia sit P18 mot diém bat ky nm trong mat phang cia hinh binh hanh ABCD. Qua A,B,C,D ké céc dutmg thang theo thit tu song song véi CP.DP.AP.BP. Ching minh ring cic duéng thing vira dyng déng quy tai mot digm Q nao dé va dutmg thing PQ ludn di qua mot digm cd dinh khi P dai ché trong mat phing. 3.8 Hinh binh hanh MN PQ ndi tiép hinh binh hanh ABCD sao cho M,N, P va Q lin lugt nim trén cc canh lien tiép AB, BC,CD va DA cia ABCD. Ching minh ring hai hinh binh hinh MN PQ, ABCD noi, ngoai tiép nhau 6 ciing tam d6i xin; 3.9 Hai ngudi choi mot trd chi \dat déng xu len mat bin hinh chi nhat". Quy tée choi nhursau: Déng xu duge phép dat vao bat cif ché trong nio, hai ngudi in lugt dat cdc déng xu lén mat bin, Ai dén lugt dima khong thé dat duce éng xu vao dau thi bi thua. Ching minh rang néu biét cach choi thi ngudi diu luén thing cudc. 39 3.10 Tit giée ABCD noi tip dudmg tron (0). Ching minh rang céc dutmg thing di qua trung digm ciia mot canh (hoge mot dung chéo) va vudng g6c véi canh doi dien (hoge duimg chéo kia) thi dong quy. 3.11 Cho duéng thing a, duimg tron (O) va mot diém P khong nim trén a va (0), Tim trén a mot diém A va én (O) mot diém B sao cho doan AB nhgn P fam trung diém. 3.12 Hay cat mot tam giéc ABC cho trudc bang ba dutmg thang song song vi cde canh ciia tam gide dé thu duge mot luc gidc (16i) ngoai tigp duge mot dudng tron, 3.13 Cho hai day cung khéng cit nhau AB va CD cita mot duimg tron (O) va mot diém P trén day CD. Tim trén dutmg tron mét diém S sao cho géc noi tiép ZASB chin trén day CD mot doan thing MN nhan P lam trung diém. ©. Vn dung phép déi xing true vao vige gidi cdc bai toan sau day 3.14 Mot dudng trdn thi ba (w) cat hai duimg trdn déng tam O Kin luot lén tren (w) & cde diém A,B,C,D. Chimg minh rang néu A, B,O thing hing thi C,D,O cing thing hang, 3.15 Hai duimg trdn bing nhau (O;) va (O2) cing tip xiic trong véi dutmg tron (O) cde diém Ay va Ay. Mot diém M wy ¥ ciia (O) duce ndi tip voi A, va As. Che doan thing MA, eat (O,) 6 cde diém B, wong ting (i = 1,2). Ching minh ring By Bp || A, As. 3.16 Mot dim M chuyén dong tren duimg kinh AB cita mot dudng tron (0), Day cung CD di qua M cat AB va hgp véi né mot gée 45°. Chimg minh ring dai luong p ‘C? + MD? khong phu thudc vao vi tri cia M trén AB (p khong adi véi moi M tren [AB}) 3.17 Cho tam gide ABC can & A mot duting thing A quay quanh A. Goi D fa diém d6i xing v6i C qua D. Tim qu tich giao diém M cita duting thing BDvad. 3.18 Tinh g6c tgo bai hai guong phang biét rang mot tia sng bat ky sau bén Lin chiéu vio ca hai gutong (méi gong dip vao hai Kin) thi phan xa theo hung nguge lai hung ban déiu, 3.19 Cho dutmg thang cry va hai diém A,B nim cing phia véi ry. Hay tim tren wong thang zy mot diém P sao cho géc ZAP e bing hai kin g6c ZBPy. 3.20 Ching minh ring a) Dign tich ciia mot tit gide Idi bat ky khong lén hon mot nita téng cla tich d9 dai cdc canh doi dign S(ABCD) < }(AB.CD+BC.DA), (*) 40 b) Datu dang thie trong (*) dat duge khi va chi khi tt gide noi ti¢p duge mot duimg tron va hai dutmg chéo vudng ge véi nhau, D. Vin dung phép quay va déi hinh vo vige giai cée bai toin sau 3.21 Cho hai duimg tron bing nhau (Q,) va (Qs). Tim tat c& ede phép doi hin thuan va nghich (d3i hinh va phan déi hinh) bién duémg tron nay thinh dung tron kia, 3.22. Tren duimg thing a va b cat nhau 6:mot diém P c6 hai dong tirchuyén dong thang déu véi cing mot van tc nhung khong gap nhaué P: My tren a va My wen b, Chimg minh ring & bat ky thoi diém nio, duimg tron ngoai tiép tam gide My MP luon di qua mot diém e6 dinh O nio d6, khdée P. 3.23. Ching minh rang hai dutmg trdn bang nhau va cit nhau shai diém thi twong ting v6i nhau trong mot phép quay ma tam quay 1 mot trong hai giao diém va cde dung thing néi ce cap diém tuomg tng trén hai dung tron thi déng quy 6 giao diém thet hai, 3.24 Gid sit P La mot im nim trong hinh vung AiAoAyAy. Qua Ar dung dugng thing Ayr vudng géc voi A»P, qua Aa dung Aay vudng géc v6i AyP qua Ay dug Agz vudng géc voi AgP r6i qua Ay dumg Aut vudng géc véi ALP. Chimg minh ring bon dutng thing Avr, Azy,Ay2 va Ad vita dung déng quy & mot diém Q ndo d6, 3.25 Cho ba diém A, C, B phan bigt va thang hing theo th tu 46, Dung hai tam gide déu BCM va CAN nam vé cing mot phia d6i véi duimg thing AB. Goi D va F Lin luot 1a trung diém ciia BN va AM. Ching minh rang CDE 14 mot tam gidc déu. 3.26 Lay cde canh cia mot tam giée ABC bat ky kam day, dumg ra phia ngodi tam giée ABC batam gide déu BCA',CAB'va ABC’. Chimg minh ring cic tim Ay, B,,C,, cia ba tam gide déu vita dung la céc dinh cia mot tam gide déu (Bai ton Napoléon). 3.27 Cho mot tam giée déu ABC. Tim qu tich nhimg digm M trong mat phing sao cho MA,MB va MC Ia d@ dai cdc canh cita mot tam gide vudng nio 6. Chuong 2 C4c phép dong dang phang § 4 Su xc dinh va dang chinh tac cia mot phép déng dang phang 1 Dai cuong vé céc phép dong dang phang a) Dinh nghia phép déng dang Mot phép bign hinh f : P + P duge goi la mot phép déng dang ciia mat phang hay van tit la phép déng dang phing, ky higu Ia Z, néu v6i bat ci hai hay he s6 ciia phép déng dang, hay néi gon hon, Phép dong dang ty s6 k duge ky higu bai Z(h). Ni mot céch ngin gon, phép déng dang phng ty) sok Ia phép bien hinh ca ‘mat phing, nhan khong céch gitfa bat cit hai diém nto cia né véi cing mot so lugng k xc dinh cho trudc. 1, phép déng dang tri thinh phép doi hink, ngk . ddi hinh [a trudng hop dic biet ciia déng dang. Tirdinh nghia cia phép déng dang, dé ding suy ra rang phép dio ngugc cila phép déng dang ti so ka phép déng dang ti sO 1/k a vist 2~1(he) = 2(1/h) hay Z(k) oZ(1/h) = Ia. Tich ciia hai phép déng dang 6 cdc ti s6 kiyha 1A phép déng dang v6i ti 86 k= hy.ka, nghia la, Za(kp) 0 Zi(hi) = Zra(hika), nhumg Z,(ki) o Za(ko) = Za(haky). Phép bign hinh déng nhat Jd 1 mot phép déng dang. b) Cae tinh chit cfia phép déng dang 41 42 Cing tir dinh nghia ca phép déng dang, ta dé dang suy ra céc tinh chat sau day ciia phép déng dang, trong d6 c6 nhimg tinh chait cla phép di hinh. Dinh ly 15. Phép déng dang bao toan su thang hang ciia ba diém va thi tw cia ching trén duéng thang chifa ba diém 46. Cu thé la: Phép déng dang bién ba diém A, B,C thang hing theo thit tu dé thanh ba diém A’, B’,C’ thang hang cling theo tha ty d6. Hé qua 1. Phép déng dang bien dudmg thang thanh mot du’mg thing bién ‘m6t tia thinh mot tia, bién mot doan thang thanh mét doan thang c6 do dai duge nhan lén véi hé s6 (ty s6) déng dang (4'B! =kAB,V{A, B}) Hé qua 2. Phép déng dang bién mot tam gidc thinh mot tam giéc déng dang véi né; bign mot géc thnh mot géc bing né; bien mot du’mg tn thinh mot dutmg trén, trong dé tam bién thinh tam con ban kinh duge nhan lén véi hé s6- (ti s6) déng dang (R! = KR) ¢) Phan loai cae phép déng dang Caing gidng nhu phép di hinh, phép déng dang chia lam hai logi, logi mot va logi hai tuy theo n6 bio toin hung hay dio ngugc hung ciia hinh. Ta goi 1a phép déng dang thudn, hay vin tat 1A phép déng dang, mot phép déng dang phang bio toin huéng cia hinh. Ta goi phép déng dang nghich hay cdn goi I déng dang guong, hoac phan déng dang, mot phép déng dang phang dio nguoc huting cia hinh, Chi thich. Phép déng dang tuy khong bio todn khoding céch gitta hai digm nhung giéng nhu phép doi hinh, phép déng dang bio toin géc (n6i ding ra La bio toin dé 1én thong thudng ciia géc, bao gém géc gitta hai tia, gidta hai vécto, gia hai dudmg thang). Vi thé ngudi ta con néi phép déng dang 1a mot phép bién hinh bio gidc. Noi mot céch chi tiét hon thi: phép déng dang phang thuan con bio toan ca hutng ciia géc, vi thé ta ndi phép déng dang thuin bio toan do én dai sO ciia gée dinh hudng (gidta hai tia, gidta hai duimg thing). Phép déng dang nghich chi bao todn d9 én s6 hoc (cing tiie 4 Ién thong thudmg) ciia géc dinh hudng (gia hai tia, gitta hai dutmg thing) nhumg lam dio huéng géc, te 18 bien ‘g6c duong (46 1én dung) thanh géc Am (dd Ién Am) va ngugc lai. 4) Khai nigm vé hai hinh déng dang Dinh nghia. Hai hinh 1 va H goi 1a déng dang véi nhau néu cé mot phép déng dang Z bién hinh nay thinh hinh kia: Z(H) =H’. Néu phép déng dang Z bign hinh } thinh H', thi phép déng dang dio nguge 43 cita Z bién H{' thinh H. Ky higu hai hinh déng dang boi, ching han AA’ BIC! AABC. Hinh27 Chui thich. Vicé hai loai phép déng dang, thuan va nghich nén dé phan bit 16 hai hinh nao d6 18 déng dang thuan hay déng dang nghich néu thay cain this. Sau day, chiing ta dé cp dén mot phép déng dang dic bigt, d6 1A phép vi ty. 2 Phép vitu ” a) Dinh nghia. ‘Trong mat phang cho mot diém O c6 dinh va k 1 mot s6 thuc khée khong cho truéc. Phép bién hinh cia mat phang bién mdi diém M thanh diém M' sao cho (Hinh 28), OM! = kOM (1) duge goi 1a mot phép vi w tam O, he va ky higu la Vi hay V(O,k). Diém O goi i sdvi ty. Hinh28 phép vi ty Vio. ih hode ngudi ta cing goi MI! phép vi tu V(O, A) bién mot hinh #1 thanh mot hinh H! ‘aon c inh MT" ela tat digm M thude hinh 24) thi ta ciing n6i H{ 1a hinh vi uy cia hinh 2! hay 1! 1a hai hinh vi tu vi nhau. Tong myc n ching minh vi ching ta chi nie lai din nghia va cde tinh chi iép vi tye ma khong. -6 bé sung mot vai tinh e i oan tigp theo 44 Mot phép bién hinh diém ciia mat phng ma chéng ta da gAp trong Thi du 2 eiia Bai (mae 3) cho ta thi dy ci tiem vé mot phép vi te phan. That vay, hay Q) trong dé G 1a trong tam cita tam gidée ABC dé cap dén trong thi du d6. Vi vay, phép bign hinh f trinh bay trong Thi du 2 cia bai tp 1 (muc 3) dé chinh Ia phép vit V(G,=1/2) tam G. ti sok Ngoai ra, c6 hai trudng hop dic iia phép vi ty, dng véi hai gif tri cia he s6 hk 1a k = tI. Néu k = +1, khi dé OM! = OM(VM). Vay trong ting hop nay, phép vi tw ti s6 1 18 phép dong nhat. New k = —1, thi OM! = OM, (¥M) tie A" doi xing v6i M qua diém O. Vay trong tring hop nay phép vi tu ti so —1 1a phép doi xing qua tim O. b) Cae tinh chit cia phép vi ty Dinh ly 16. Phép vi tw ti phép vi tu V(O, k) bign Mth: mot phép déng dang ¢ NV thinh N’ thi M'N Chi thich. Hinh29 (i) Nguidita cdm néi, phép vi ty ti s6 & bign mot vécto 7 thanh vécto v" bing k lin vécto @, nghia fav nhan mot véeto len k lan. =v’ hose n6i gon hon [a phép vi ty ty sok (ii) Vi phép vi ty 8 mot phép déng dang dae bigt nén phép vi tut c6 moi tinh chat ciia phép déng dang. Tuy nhien, phép vi turcdn c6 nhiing tinh chat dae trung riéng ciia né. 45 Dinh ly 17. Trong mot phép vi ty, tam vi ty La digm bat dong duy nhat va mot dudng thang khong di qua tam vi tu bién thinh mot duéng thang song song ‘véi nd. Chim dutng thang 6 tam 6 tam vi tu [a tap hop nhiing dutmg thang bat bign duy nhat cia phép vi ty. Phép vi tur phang gay nén mot phép vi tur trén moi duéng thang (bat bién) di qua tam vi ty. Dinh ly 18. Moi phép vi ty phing ti s6 duong hay am déu 1A mot phép déng dang th Chii thich. He s6 (hay ti s6) vi tr & 6 thé dwong hay am. Vi thé, wy theo k duong hay Am, ngudi ta goi phép vi tu V(O,k) 12 phép vi tw duong hay phép vi tu am. Dé thy rang phép vi ty duong bio ton huéng cia véeto con phép vi ty am thi déi hudng cita yécto (Hinh 30). Tuy nhien, dé thay ring: V(0,—k) = 'D(0) oV(0,k) = V(O,k) o'D(O), trong dé k > 0. Tir dé suy ra phép vi duomghay 4m déu bao toin hung cila géc dinh hung gitta hai vécto hay gia hai dutmg thang va do d6, bio toin hung cia hinh, Hink 30 minh hoa tinh chat nay. Hinh 30 ©) Tam vj ty eda hai dudng tron Dinh ly 19. Phép vi ty bién mot duimg trdn thinh mot dudng tron. Dio lai, néu (Oy, Ry) va (Oz, Ry) [a hai dudmg trdn phan bigt elia mat phang thi, néi chung, e6 hai phép vi ty bign dutng tron nay thinh dudng tron kia ma tam vi tr 1a cée diém chia trong va ngoai doan néi tm theo ty s6 hai ban kinh, Goi $ 1a tam vi ty, k 1 he s6 vi ty, The thi, néu S = O thi phép vi ww V(O,k) bign duimg tron (O,R) thinh dug tron (O,|A[R) déng tam O, bin kinh R!=|A.R. Néu S 4 O thi V(S,k) bign dudng tron (O, R) thinh dutmg tron (O, 2’) trong = i 6 O! duge xc dinh béi SO’ = kSO va R’ = [RIP cling tiie 1a S chia O'O theo Dio lai, gid sir(O;, R;), i= 1,2 1a dudng trdn khong déng tam va ciing khong cing bin kinh. The thi ton tai hai va chi hai phép vi tw V(Sishj)o4 = 1,2 tam Si va Sa theo thi ty chia ngoai va chia trong doan ndi tam O20; theo céc ti s6 duong twong ting ky = Ro/Ry va ke = —Ry/R; bién (Oy, Ry) than (0, (Hinh 31), Cac diém $, va Sp theo thit ty dyoc goi 1a tam vi ty ngo’ tur tong ciia hai dutng tron (O,, R;). Cé hai trudng hop dae biet a) Hai dudng tron déng tam nhung khong cing ban kinh: O; = O2=O,Ry 4 Ro. Khi 46, cd hai phép vi ty cing tam S tring véi O va ti s6 vi ky = £Ry/ Ry déu bién dutmg tron (O, Ry) thinh dudmg trdn (O, Io)» b) Hai duing trdn khong déng tam nhung cing ban kinh O; 4 Oo, Ry = R R (cing tic hai during trOn bang nhau (O;, R) va (02, R). Trong trudng hgp nay, phép d6i xting tam O 1 phép vi ty duy nat (c6 he sé k = —1) bién (01, R) thinh (02, R), hoac nguge Iai, trong d6 O 1A trung digm doan 0:02. Hinh 31 Phép doi xting tm O Ia phép vi tur dae biet tm O ti s V(O,-1). Chi thich. a) Néu hai dung tron tigp xtic nhau & A thi tigp diém A cita ching [a mot trong hai tim vi ty Si, = 1,2. A 1 tam vi ty trong hay tam vi ty ngoai cia hai dutmg tron (1, Ry) va (Os, Ro) tuy theo hai dutng tron d6 tie xiic ngodi hay tiép xtic trong v6i nhau é digm A (Hinh 32a va Hinh 320). Con tam vi ty thi hai I& diém chia ngodi hay chia trong [020] theo ti s6 +Ro/ Rye b) Néu hai dung trdn khong dung nhau, hode ngoai nhau thi giao diém cia (0,02) va mot tigp tuyén chung ngodi hoae mot tigp tuyén chung trong cia chiing cling x4e dinh tam vi tur ngodi Sy ho uf trong Sp ciia hai dumg tron (O1, Ry) va (Ox, Ro) Hinh 33a va Hinh 326). 4 Hinh 32a, Hinh 32b Hinh 33a, Hinh 336 4) Tich cia hai phép vi ty Dinh ly 20. Tich cla hai phép vi tu c6 ti s6 ky va hy LA mot phép vi ty. TH 86 k = hiky 6 tam thang hing véi tam ca phép vi ty dé hoge mot phép tinh tién tay theo kiky £1 hoae kika =1 Chitng minh. Gid sit Vi(Oy,kx) va Va(O2, hy) la hai phép vi tu. D& thay rang néu O; =O, = O thi Vo(O, ky) 0 Vi(O, kx) = V(O, kiky) néu kik, A 1 Néu Or 4 On, luge dé ciia tich fa MY My“ M'. Theo dinh nghia, véi moi M, thi O; My = k1OiM va OM" = kyOoMy (Hinh 34), Khir diém trung gian M, nhé quy tac cong vécto Oo, = O1M, = —O102 ‘Hé thite trén tré think VM Oo = kefk\O,M — 0,03} a) Vay digm O phai tim, néu tén tai thi duge xéc dinh bai (1) [bang ech cho M'=M =O; = ka{4O,0— 0,02} 2) 48 nghia Ia (1=hyky)O:0 = (1k) O02 2) Hinh 34 Trung hop téng quat kia 4 1. Dang thie (2) xde dinh mot dim O duy hat, thing hing v6i O;,0». Bang cach lay (1) trit (2) vé doi v6, ta di dén VM, OM'=kykOM @) Hé thite nay x4c dinh phép vi ty V(O, knkz) tam O ti s6 k= kika(# 1). ‘Trudng hop dic bigt kyk2 = 1. Khong c6 mot digm bat dong mio, nhung véi hy = 1/ky thi dng thite (1) tro thanh YM, O2M' =O;M = 10,02 ta viet lai dudi dang YM, MM’ =(1—k:)OiO, (4) He thite (4) nay xc dinh phép tinh tign theo vécto “v" = (1,h2)O,02. Day la digu phai ching minh, 3 Su xéc dinh mot phép dong dang phang Dinh ly 21. Cho ABC va A'B'C’ Li hai tam gide déng dang cho true trong mat phing P- AB! _BIC!_C 4B 7 BO Ga W* 49 Bao gid cing c6 mot va chi mot phép déng dang Z : P +P bién A,B,C theo thifty thinh A’, B',C! Chiing minh. Xétphép vi ty V(A,k); né bign tam gidée ABC thinh tam gide AB,C, twong d6 AB, = kAB, kCA (Hinh 35). Nhut vay phép vi tw V(A, k) bign tam gide ABC thinh tam gite AAB,C; = AA'B'B!. Theo dinh ly 3 thi c6 mot phép dai hinh D duy nhat bién A, By, C; theo thirty thanh A’, B,C! tich DoV(A,k) 18 mot phép déng dang Z((h|) ti s6 |k| bién A thanh B thinh B! va C thinh C’ Hinh 35 Bay gid gid sirc6 hai phép déng dang Z, va Z) déu bien ABC thinh A’B'C! thi phép Z; ‘0 Z Ia mot phép dong dang ti s6 1, tic 1a mot phép doi hinh D bign tam gide ABC thinh chinh n6, trong d6 bién A thinh A, B thanh B va C’ thinh C. Vi vay, theo dinh ly 2, Z;!0Z, = Id va do dé Z, = Z;. Dinh ly da duge ching minh, H@ qua 1. Mot phép déng dang phing bao gid cling c6 thé phan tich duge thinh tich ctia mot phép vi ty va mot phép ding cy (ddi hinh hoge phin dai hinh) theo thi tu dé hay theo this ty ngugc lai. Chi thich, a) Doi véi cée phép dang cy thi phép d6i xing - true déng vai trd phiin tir sinh cdn d6i véi céc phép déng dang thi déng vai trd phan tir sinh 1a phép vi tur va phép d6i xcing - truc, trong d6 phép vi tu déng vao trd ndi bat, phin biet sut khde nhau gitta Z va D 6 ti s6 [al 41 b) Néu Z(lAl) =DoV(A,k) thi Z-"(Qa)) = (Al/k)oD™' =Z([z)) cc) Suphan tich néi trén IA khong duy nhat. Hé qua 2. Mot phép déng dang thuan trong mat phang tuong img véi tich cita mot phép vi ty va mot phép quay hay mot phép tinh tien (theo thif tu dé hay theo thi ty nguge lai). H@ qua 3. Mot phép déng dang nghich trong mat phang tong (ng véi tich ciia mot phép vi tu va mot phép doi xting - truc hay mot phép doi xing ~ truot (theo thit ty dé hay theo thi ty ngugc lai). 4 Diém bat dong va dang chinh tic cha mot phép déng dang phang a) Dang chinh tac cia mét phép déng dang thuan Dinh ly 22 va Dinh nghia Moi phép déng dang thuan trong mat phing khéc vvéi phép tinh tign déu c6 mot diém bat dong duy nhat O va tuong duong véi tich giao hoan clia mot phép vi ty va mot phép quay cing tam O. Tich giao hodn nay duge goi 12 phép vi ty ~ quay va 1a dang chinh te cla phép déng dang (thye sur ) thuan trong mat phang. Chiing minh. That vay, n&u Z Wi mot phép vi ty V thi tam vi 1a diém bat dong duy nhat ciia n6. Gid sir ZV va gid sit O 1a diém bat dong can tim cla phépdéng dang thuan Z duge x4c dinh bai hai tam gic déng dang (thuan) tuong tng ABC va A'B'C', wong d6 A’, A; B!, B;C',C 1 cée cap diém twong ting. Vi 2 AV nén A'B' | AB (Hinh 36), Ti AOA'B! = AOAB ta duge ZOAP =ZOA'P, ZOBP = ZOB'P, ZAOB= ZAOB' trong dé cdc géc xété day déu la gée dinh hutng (mod 7) clia cdc dutmg thing. BBIOP 1a hing ta gidc noi tip. Vay diém bat dong, iia hai dudng tron (AA'P) va (BB'P). Déng thai, tir dang the (OB',OA') = (OB, OA). theo hé thite Sale, ta duge (04,04) =(OB,OB’)=¢ (mod 2m) a Ngoai ra, cing tit AOA'B! » AOAB, tacin c6 OB AB OA” OB” AB Vay, cde dang thite (i) va (ii) cdn néi lén rang mot phép déng dang thuan Z trong mat phang bao gids ciing phan tich dyoe mot cach duy nhat thinh tich giao hoan cia mot phép vi ty tam O ti sO k va mot phép quay gée y (mod 2) xung fa digm bat dong cla Z. k Gi) fe A 11 ding dang k #1 vado ds Z¢D 51 Tich gid hodn nay duge goi Ia phép tu quay va gée y cling duge goi A g6e déng dang. Phép déng dang thuan tam O géc y va ti sO k duge ky higu 1a 2(0,¢,h). Nhu vay, 2(0¢9,k) =Q(0, 9) V(O,k) =V(0,k) oQ(0,9) Hinh 36 Chii thich. Tren hinh 36 ta khong vé cap diém tuong tng thé ba C.C Vithye ra chi edn hai cip digm tuong ting A,’ va B, B' Ia dii dé xc dinh mot phép déng dang phang. ‘Tuy nhien, cin dén mot cap diém thit ba C,C' nita dé dinh hung, ciing t ic dinh Z 18 thugn hay nghich tiyy theo hai tam gide déng dang ABC va A’BIC' IA ciing hung hay khéc huéng. b) Dang chinh tae ciia mt phép déng dang nghich (phing) Dinh ly 23 va Dinh nghia. Moi phép déng dang nghich trong mat phing khong phai 18 mot phép doi hinh nghich déu c6 mot dim bat dong duy nhat O va tuong ting vdi tich giao hodn cila mot phép vi tyr duomg tam O ti so k va mot phép doi xing qua mot dung thang A di qua O (hoje mot phép vi ts i sO —k va mot phép dai ximg - true A’ ciing di qua O va vuong géc voi A 60). Tich giao hodin nay duge goi Ia phép vi ty - d6i xing va true doi xing A duce go1 i truc déng dang, O duge im déng dang. Phép dng dang nghich tam. O, true A va ti s6 k duge ky higu i Z(O,A, A). ne? ‘minh. That gid sit Z #D(A),Z D(A, 7") vasiiisliO B dim 52 tam gide OAB nén ta c6 (Hinh 37) (OA',OB') = -(04,0B) aw Hinh 37 Tir d6 suy ra hai cp dutng thing OA,OA' va OB,OB' cé chung nhau hai dugng thang phan gidc déng thdi cing truc déi xting Os,Os’, vung géc v6i nhau GO. Goi P va Q Lin lugt 1a ta duge jiao diém ciia truc Os v doan thing 44! va BB! PA’ _ OA’ _ OB’ _ QB’ PA OA OB QB = Do.(A) va By = Dos (B) va néu AB |f Os thi (A’A,) 9 B'B,) =O. Tir dé suy ra diém bat dong O 1A giao diém cita hai trong ba dung thang déng quy: PQ,A’A; va B = (PQ)N(A’Ay) = (PQ)N(B'B,) = (4A) 9 (B'B,) Chai thich, a) Néu dé y ring vik 1 nén khong nhimg (44’) va (BB’) cit Os tai P va Q ma (AA’) va (BB') con cit ca Os’ theo thit ty ede diém P’ va Q! vata cing he thc (2) tong 46 P va Q duge thay déi béi P! va Q', chi khéc 12 trong tutng hop nay thi P’ va Q! déu chia ngodi céc doan A’A va 2) BB theo ti s6 sO hoc k. Boi vay, diém bat dong O cin 1A dim déng quy cia bén dutmg thing PQ. P’Q!.A' A; va B'B,, trons d6 va P',Q! theo thét w [A] va[B'B|theot 6 hoe hy trong d6 k Wa ti 5 déng dang cia A’ B'C! ‘ déng dang cia Z nghich, 53 b) Tren day Ia truing hop A'B! | AB. Ta cdn phai tudmg hop AB" || AB. Néu A’B' | 2 el dling thoi dung thing PQ di qua giao diém cia AB! va A'B, Giao diém nay chinh 1a digm bat dong O ciia Z nghich cin tim (Hink 38): O = AB'N4’B € (PQ). (PQ) chinh ld truc Os cita phép déng dang nghich. Hinh 38 Cang tir ing thiéc (1) va (2) ta suy ra phép déng dang nghich Z trong ‘mat phéng khong nhiing c6 digm bat dong duy nhat O xac dinh nhurtrén ma con phan tich duroe mot cach duy nhat thanh tich giao hon cia mot phép vj tu tam O tis6 k (hoe —F) va mot phép doi xting tryc A di qua O (hoae A’ wuong géc v6i A 6 0). Tren Hinh 38 (PQ) = A,(P'Q’) Tich giao hodn nay duge goi Ia vi ty doi xiing va ky higu van tit & 2(0,A,k). Hai dang 2(0,g,k) va 2(O, A.A) ~ phép vi tu - quay va phép vi tur d6i xiing goi 1a hai dang chinh tac cila phép déng dang phang. 5 Bai tap vé xc dinh va dang chinh tac cua mot phép déng dang phang 4.1 Chiing minh dinh ly: Trong mat phdng cho hai doan thing AB va A’B' kAB, (0 P 1 mot phép bien hinh cha mat phang. b) Hay goi tén phép bien hinh d6, § 5 Van dung phép dong dang vao viéc giai mot so bai toan hinh hoc phang = Thi du minh hoa va Bai tap 1 Ung dung phép vi tu vao toan hinh hoc ‘Thi dy 1. Ching minh ring trong mot tam gi a) Ba dumg trung tuyén déng quy & mot diém, diém nay céch mai dinh cia tam gide bing 2/3 do dai ciia duimg trung tuyén phat xust tir dinh d6, b) Trong tam G trye tam H nO dudng trdn ngoai tip mot dung thang, goi IA duémg thang Ole, ng nim trén Ching minh. a) That vay, theo két qua néu trong ba tam giéc ABC inh 39) trong phép vi tr s6 k= BC /BC =-1/2 = = gide A'BIC', nghia 1a ta duge (Hinh 40) GO = —4GHH hay la GO Day 18 digu phai chimg minh Hinh 39 ‘Thi dy 2. (Dinh ly vé dudng trdn Ole) Chimg minh ring trong mot tam gidc, trung diém c&c canh, chan cdc duting cao va trung diém cia céc doan thang néi trye tim véi cde dinh cing nam trén mot dutng trdn, goi fa dutmg tron chin diém hay dung trdn Ole cita tam gid 46. Ban kinh dung tron Ole bing nia ban knh dung tron ngoai tigp tam gide va tam O' cia dung trdn Ole thing hing v6i tam O dufong tron ngogi tigp, trong tim G va trye tam H trén durdng thing Gle sao cho O’ fa trung diemé cia doan thing OH va bon diém 77,G,0,0! lam thanh mot hing diém digu hoa. Hinh 40 Ching mink. ‘That vay, goi O! 1a tam dutmg won (A'BIC') the thi O! 18 hinh vi tw eta tam O dutmg tron (ABC) trong phép vi ty V(G,—1/2)- Do 46 O! cing nim tren 57 dudng thing Gle OGH va ta c6 (Hinh 40). GO’ Tir he thie tren ta dé dang suy ra 7G OH. Vado dé ta duge (HG00) = Z20H Day 18 digu phai ching minh. Cing tird6 suy ra HO __Go_1 a HO hay 1 HO! = 4/70 va do d6, H fa tim vi tw thé hai bin duimg tron (0) = (ABC) thinh dudng tron (O") = (A'B'C') theo ti s6 1/2. ‘Trong phép vi ty thét hai V(H,1/2) nay, cde dinh A,B,C ciia tam gide ABC theo thit ty bign thanh céc trung diém A",B",C" ciia céc doan thing n6i tryc tam H véi cdc dinh A, B,C va dods, nay nim trén dutmg tron (0)=(A'B'C). Ngodi ra, dé thay ring A’A",B'B" va C'C" Ia cde duimg kinh cha dudmg tron (0'), Tird6 suy ra (0!) ciing di qua chan Ay, Bi, Ci ca cae durtmg cao AAy, BB,,CC; cia tam gise ABC. Chinh vi vay, dung trdn (0") durge goi la dung tron chin diém hay dung tron Ole cha tam giée ABC. « 58 Hinh 41 Chitng minh. Trade hei, ta ky hiGu nhut sau cho gon Vi = V(C',hx),Vo = V(BIka)sVa=V(A', ka) trong d6 a ‘Thé thi 16 rang ta 6 két qui nhursau Vii BH A, V2: AV C,V3: BC Nhung theo dinh ly 20 vé tich ciia hai phép vi tur thi Vo Vj 1a mot phép vi ty 66 tam thing hing véi C' va B’, va c6 ty s6 vi wr kha £1 (i BIC’ (BC) va (BIC) N(BC) = A’. Lai do V2 0 Vi bién B at neo nrieh Yoo phai la giao diém A’ cita hai dutmg thing BC va BIC’ N6i khde di lA V3 0V: ay, ky = hy, hay 1a ky.ko.by! = +1 (2). Cudi cing, thay cde gid tri cia ki(i = 1,2,3) tir (1) va (2), ta thu durge (#) can tim, Thi dy 4. Ching minh ring quy tich nhing diém ci th tir d6 dn hai duimg thang cat nhau & mot i gdm hai dudmg thing di qua O. iém O bang k(k > 0) Hinh 42 Ching minh. Gia sit «'x va y’y IA hai dutmg thing cho tude cit nhau & :m O (Hinh 42) va M 1a mot diém cia mat phing sao MH/MK = k (hing sO duong cho trude), Goi M; 1a diém twong tng cia diém M trong mot phép vi tu thay déi®! tam O. Vay qu tich nay sé gém nhiing dutng thang da qua O. Digu do khign ta nghi dén viéc tim nhiing diém thude quy tich can tim trén mot cat tuy¢n AB cia hai dutmg thang O:r va Oy sao cho OA = OB. V6i mdi diém Tachi cé thay, vé gid tH clia ti sO vi tw ma thoi 59 g6c nhon bing nhau & MA/MB. Diém M M cla AB cic tam gis ‘Ava B thi déng dang mot diémciia quy tich néu ty so. MA/MB =k, 1a MA/MB =k. Khi ty so digu kign ddi hoi cia bai togn. Vay quy tich cn tim bao gém hai duting thing (OM) va (OM’) lign hop digu hod d6i v6i hai dung thang Oz va Oy da cho (vi hanh digm (ABMM') 1a digu hoa. wong MAH va MB Thidu5. Tam giée ABC noi tip duimg tron (O) cho truée c6 hai dinh A, B 6 dinh. a) Tim qu tich trong tam G ela tam gide ABC. tryc tam cita tam g Loi gid. a) Goi C, la trung diém canh AB, ta 06 (Hinh 43) C,G = 4C,C. Suy ra {G} han duge tir {C'} = (O}, trong phép vi ty Vi(C., 1). Vay {G} li dudng tron tam On bin kinh Ry = R/3, trong dé R Ia ban kinh ciia (O) cdn O; duge xéc din béi OO! = 20€,, (1) Hinh 43 b) Tacé OF = 30G (Thi du 2). Do d6, {H} duoc suy ra tir {G} boi phép vi ty Va(0,3) tam O, ty s6 k= 3. Vay {H} a dutmg tron(O!, R), trong a6 0! duge xée dinh bai OO! = 300,, hay (theo (1)) OO! =20€,,. (2) He thiic (2) cdn chiing ta ring O! doi xting v6i O qua 6 ta di dén két luan ring quy tich true tam HT cita tam gi (O',R) doi ximg véi duimg trdn (O, R) ngoai ticp ABC qua canh AB. Nhén xét. Theo thi {H} duge suy ra tir {C} = (O,R) bai phép déng dang Z = Vs 0V; IA tich ca hai hép vi ty Vi(C,,1/3) va Va(O,3) * . vécio OO! = 20C;, (khong déi) theo Dinh ly 20. Ta lai thay ket qua da biet lui 60 = oo (CH = 20€,) nén trong thi du 2, myc 1 cia Thi dy 6. Dung mot duimg trdn tip xiie v6i mot duimg trdn (O) cho tube VA v6i mot duting thang A cho trade, biét mot trong céc tip diém. Léi gid. Gid sit (w) 12 mot dug trdn tiép xtic v6i dudng trdn (O) & A va v6i dutmg thingA & B (Hinh 44), Ta nghi dén mot phép vi tu tam A [a tiép di a (O) va (w). That vay, (O) [anh cita (wv) trong mot phép vi tr tam A bign B thanh mot trong hai véi A. Ta xét hai trudng hop nh bai t 1 nit Chose D cita dung kinh CD cia (w) vudng géc in da dat ra (Hinh 44). Hinh 44 1) Néu diém A cho truée trén dung trdn (O), thé thi chang han, dutmg thing CA cit A 6 diém B va do d6, phép vi ty V(A,k = AB/AC sé bien dutmg tron (0) thinh mot dutmg tron (w) tigp xi voi (O) & A va voi A 6B. (MOUId gidi thithai duge thy hign véi dutmg thang D.A). 2) Néu diém B cho truée tren duimg thing A thé thi dutmg thing CB cit lai (0) & phép vi ty V(A,k = AB/AC) s& cho mot duting tron (w) tigp xiic vi D 6 B va v6i (O) & A. (MOt Idi gidi thet hai thye hign tuong tu véi (DB), 2 Ung dung phép dong dang thuan (vi tu quay) vao toan hinh hoc ec gid Thi dy 7. Xé mot tam giée ABC va mot diém D cho trude. Ta dung tam gide ADE, DBF déng dang (thuan) véi tam giée ABC. Hay so sinh tam giéc ABD, ACE va CBF vatim higu bin chat cia té gide CEDF. Loi gidi. Tu AADE » ADBF ta suy ra AABC ~ AADE (Hinh 45) va cing vay, AABD » AACE. Céc tam gide BAC va BDF 1a déng dang thuan, tir 46 cing 61 suy ra ABAD & ABCF. Vi dang thuan. ay, ba tam giéc ABD, ACE va CBF Ia déng Hinh 45 Nhén xét. Tit ba tam giée ABC,ADE va DBF déng dang thudn ta da thu duge ba ta tam giée ABD, ACE va CBF cing déng dang thuin, Nhu vay 1a ta 6 thé tra0 di vai trd cia C va D cho nhau réi 46. Bay gid ta dat (AB, AC) = a (mod 2r) va AC AB. Thé thi phép déng dang thuan Z(A,k.@) s& bién BD thanh CB, cdn phép déng dang thuan 2(D,k.a) sé bién BD thinh FD. Tird6 tacé CE = kBD va FD =kBD suy ra CE = FD (1), Mat khéc (BD,CE) = a va (BD,FD) = a suy ra (CE,FD) = 0, (mod 2n) (2). Tir (1) va (2) suy ra CE = FD va do dé, CEDF ti mot binh binh hanb. ‘Thi du 8. Hay sit dyng phép déng dang thuan dé giai bai tap 3.26 (Bai toan Napoléon: Vé tam gic déu A,B,C, sinh bai mot tam gide bat ky ABC) Dé thay ring A,BC, By g6c & day 30°, Ky higu Z, 2,02; Dé thay Z; 2,02; 2 Bot Covi a C,AB 1a ba tam gi 2(C,30°. V3) Zo 2A A Bot Ava Zp: Alo Ag? Jo Ay. Matkhée, kiky = v3.25 cin déng dang 6 Xét teh AF C,, Do d6, pity, = 2.30° = 60° 62 nén Zo Z; 1 mot phép ddi hinh D, cy th Cy. Vay AgBoC la mot tam gi phép quay tam A, bign B, thanh déu. Bai toin duge chtmg minh. ‘Thi du 9. Trén hai dutng thang a va b cit nhau & mot diém C cé hai dong tir chuyén dong thang déu nhung véi van te khée nhau: A tren a véi van te Ui. B wen b v6i van te t, v1 # v2. Ching khong gap nhau &C. a) Chimg minh ABC ciing lu6n di qua m6t diém cé dinh O nao dé, khic C. b) Tim quy dao chuyén dong cita dong tir M luon & vi tr trung diémcita [AB], A(t) + B(t) tirdutmg mot nh xa déng dang (thuan)Z. That vay, gid sit . Dé cho gon ta ky higu nhur ByBy _ vslta—th) _ 2 =k, (0 < k khong di) va (Ay Ao, (B)B2) = v (mod 2m), trong d6 (a,b) =~ (mod 7) Ia gée gitta hai duémg thang a vi b. Boi vay, trong mat phing c6 mot phép déng dang thuan 2(O,¢,k) bién a thinh 8, trong 46 diém A(t) bien thinh diém B(t) (Hinh 47). Hinh 47 ‘Theo Dinh ly 22 thi tam déng dang (digm bat dong) O ciia Z 18 giao diém thit hai ca hai dudmg tron (A,B,C;) va (ABC). Vay duimg trdn (ABC) luon di qua O, khéc v6i C'e6 dinh. b) Ky higu A, = A(t), Bo = B(ta= 0). My 12 trung diém celia doan [A,Bols U1 Va ty Ia cde vécto van t6c cila A va B. 63 Tra lai. Quy tich cia M 1a duimg thing Mm di qua M, va 66 vécto chi phuong 1a 2 MoM = (a -+03)t) 3(0) +02) (suy ra tir ‘Thi dy 10. Dung mot tam gide bigt cc dO dai ha, hp, te ba dung caociiand. Phan tich. Goi a,b,c la d6 dhi céc canh clia tam giée ABC phi tim, ta 06 1 11 hgh, sbiow bb Aa tate, BOT a Te Tha Tethys e Cie ding thie (*) ching ta ring néu t6n tai, thi tam gide cin dug sé déng dang ¥6i mot tam gife T €6 d0 dai esc canh al W.c, ky higu Ta, 0.) v6i a’ = #,b =h, vad = hy ) Vay, vige dumg tam giée ABC phai tim dua vé vige dung tam giée T(4’B'C’) €6 eéc canh xc dinh bai (**): AABC o AT(A'B'C') Tuy nhién, vige dung tam gide T Iai rat don gin béi vi lai dua vé viée dung doan ty Ie thet tur a’. Tir su phan tich trén day, ta suy ra timg buéc dé dung duge AABC cin tim, Bude dung cudi cing la dung tam giée ABC, vi ty véitam gide ABC, (Hinh 48), trong phép vi tu V(A,k) tm A, ty s6k = #2, trong dé AB,C 1a mot tam giée dung bat ky, mién Lk AABSC, - AT(a,Wc) va hi, = AH. 1a chigu cao AH, ha tit A cia tam giée AB,C,. Dé ding chimg minh tam gid ABC 06 cc chigu c20 18 Rash Hinh 48 Bign ludn. Bai ton c6 Ii gid hi va chi khi ton Boi vay, bai todn c6 nghiém khi va chi khi ZL, mot tam gide nio dé; cing te 1a tén tai "Gein Lee bigu thi d6 dai ede canh eita ie he 64 canh cho trudéc. Thi dy 11. Ding mot ut a,BC=b,CD =e,DA (Idi) noi tiép ABCD bikt do d d, trong dé a,b,c LA nhiing do a Phén tich. Gi sit ut giée ABC noi ti¢p khi va chi khi ZA-+2C ch tim da dung duge. Tit ZDCB. Tren Cx lay diém E sao cho ADCE = ADAB. noi tigp ABCD duge quy vé vige dung ADC. Hinh 49 Gi sit ADCE = ADAB. Hai tam gide nay chung dinh D, boi vay ADCE duge suy ra tir A DAB bai phép vi tu - quay 2(D,,k) v DA,DC),k fd). Tanghi dén viee sit dung phép vi ty - quay dé gidi bai todn dung hinh nay la vi vay. Boi vay, dat c/d = k,(DA,DC) = ZADC =6 = ¢ réi xét phép vi tu quay 2(D,6,k). Z bign D> D, Ars C va B+ Bisao cho C'€ [BE] (Hinh 49), Thé thi ADCE » ADAB va do dé ZDCE DABva B,C,E thang hang theo thif tu d6, déng thdi ta duge 2BDE = ZADC Nhu thé 18, bing céch sit dung phép vi ty quay Z ndy ta da chuyén bai ton dung ti giéc noi tiép ABC'D vé bai todn dung tam gidéc DBE c6 cic yeu t6 da biet c ac bd DE_e sa vadodé, BE= » CD=c, DB7a BC =b,CE a a Bai todn quy vé dung diém 1) 1A mot trong cdc giao diém ciia dudmg tron 74(C,c) va duéng tron Apolonitiyt (72) 6 dutmg kinh IJ ma I va J chia trong va ngo’i doan [BE] theo ty s :/d. Dinh A dugedyng sau cing. Bign ludn. Bai todn 06 thé c6 (q1) va (2) ¢6 cit nhau hay khong. Idi gidi hoge khong c6 Idi gidi ndo tuy theo 65 3 Ung dung phép dong dang nghich (vi tu - déi xtmg) vao giai toan hinh hoc Thi du 12. Trong mat phing cho hai tam giéc déu ABC va A'BC cing huéng, ¢6 dinh C chung sao cho A’va B khong tring véi tam O duimg tron (ABC), Goi M va N lin luot 18 trung dim cia cde doan thang A’B va AB’. Chimg minh ring a) Tam gide OB'M déng dang ng b) Hai géc ZA'OB! va ZMON cé chung nhau duimg phan gide Léi gidi. a) Xét phép 2(0,3,2) = V(A,4) 0 Q(C,3). Dé thay Z : Al B's N05 0! 0 vado dé, O li tam dong dang cia Z. Tit dé ta duge OA’ = 20N va (04,0. /3. Suy ra tam gideOAN wuong & N va Lanta cia mot tam gidéc déu. Chimg minh tong tu ta c6 tam gide OB'M vuong & M va cling [a nita ciia mot D déu khéc. di tam gide OA'N. Hinh 50 Két ludn. Tam giée OB'M déng dang nghich véi tam gidée OA'N. b) Tir(1) suy ra ZA’OB! va ZMON c6 chung nhau dudng phan gide. ‘Thi dy 13. Trong mat phang cho tam giée ABC. Mot dutmg tron (O) thay déi di qua A, khong tiép xtic véi cdc dutmg thang AB, AC va cé tam O chuyén dong trén dutmg thing BC. Dudng tron nay cit tai cde dutmg thing AB va AC lin luot 6 M va N. Tim qu tich truc tam H ciia tam giéc AMN (Dé thi HSG toan quéc, Bang A, 3/2002). Hung in gidi. Gia sit 2A A 90° (vi néu / Léi gidi 1. Goi D 1a diém xu ‘Thé thi M va N theo thitty chinh Ia va (AC) va do dé trye tam HT ctia 90° thi H = A véi moi (O) digm A tren dutmg tron (0). iu (vudng g6c) cita D ten (AB) digm doi xting v6i D qua trung diém 66 MN. Goi M'va N' lin lugt 1a hinh chiéu ring tam gié ia H trén (AC) va (AB). Dé thay AHM! déng dang nghich véi tam giée ADM, Tit d6 ta duge (AH,AM')=-(AD,AM) (mod 7) Tp = yr = #8ZBAC = |c08 ZBAC| trong 46. a = ZBAC va do dé 44 = 2|cos ZBAC| = cosa]. CA trén néi lén rang (AH) di xitng voi (AO) qua phan gide Ap cia g6c ZA cla tam giée ABC va 44 = k khong déi, trong dé k = 2 cosa]. Vay H 1A inh cla O trong phép vi ty ddi xing Z(A, Ap, k). dang thie Hinh 51 Tré li, Néw kg higu (BC) = a thi {7} 1 dudng thing a’, anh cia a trong phép dong dang nghich (vi tu = d6i xing) Z(A,A = Ap,k = 2JcoAl), ba di hai diém H, la nh cla Oi =1,2) wen a = (BC) & dé ZBAO, = ZCAO) = 90° (Hinh 5.1) Chi thich. {H} =a! diquahai diém Eva F trongdé B= PE 1 AC, F QF © AB va P = D(A),Q = D(A) Dudng thing (PQ) cling duce suy ra tir a= (BC) qua phép vi w V(A,2). Lai gidi 2. Goi P = ‘D(A),Q = D(A), = PE 1 ACF =QF 1 AB. ‘Thé thi O € (BC) hay la D € (PQ) (i), Tacé MH cig hung va bing DN, DN song song véi PE, MD cing huéng va bingH.N, HN song song voi QF. Suy ra D € (PQ) hay 1 FM QD DN WA FP QP PE PE hay i H € (EF) (ii). Tir (i) va (ii) suy ra {O} {H} =(EF)\{H1,H2}; (BO)\ {01,02} ha = AO; N(EF)strong 46 F va F tong tng 67 tam cia tam gidc AF'E vudng vatam gidc AFQ vudng SE) (i ¥ j {i,j} = {1,2}) 4 Bai tap van dung phép déng dang vao viéc gidi toan hinh hoc phang A. Vin dung phép vi tu va viée giai cc bai toan sau day 5.1 Goi Ay, By,Co, Da, léin lugt la trung diém ede canh AB, BC,CD,DA ci mot hinh vuéng ABCD va P 1 mot diém bat ky ciia mat phang. Ching minh ring céc diém Ay, By,C1, va Dy Adi xiing v Ap, BoC Va Do, Cling 12 cae dinh cia mot hinh vudng. AyAgA, noi tip dudmg won (0,2). Goi B, 18 trong ta AjAgAsi 4}. Ching minh ring By ByByBs noi tigp mot dug tron; hay x4c dinh tam O; va bin kinh Ry dutng tron d6, 5.3 Cho mot hinh binh han chinh xde hom Ia, 46 dai ede canh khong d 2 dinh nhutmg céc dinh C' va D thi chuyén d6ng trong mat phang. Tim quy dao chuyén dong qua tim O hinh binh anh \e6 khép noi" 46. 5.4 Cho ba diém A,B,C thang hang theo thy 46. Goi (v1),(v2) theo tit tu 1A cae dutmg tron duimg kinh AB va AC. Mot diém M chuyén dong tren (vy), dudng thing AM cit lai (vy) & diém N. Tim quy tich giao diém P cla BN vaCM. 5.5 GiA sir ba dutng tron (A,),(Ba) va (C2) c6 cd inh, theo thi tu tiép xiic v6i hai canh cia cc g6c A,B va C' ci BC. Goi (Do) 18 dudng tron thé tuur tip xtic ngodi véi cd ba dutmg tron néi tren. Ching minh ring tim Da thing hing véi tam cdc dung tron noi ngoai tip tam gide ABC. 5.6 Dudng trOn (J) tip xiic trong véi dudng tron ngoai tigp ABC can & A, déng thoi tigp xiic v6i hai canh AB va AC & M va N. Ching minh ring trung diém ciia doan thing A/V 1a tam duéng tron noi tgp tam gide ABC. (Bé thi Toan que té IMO, Rumani 1978). 5.7 Ching minh ring néu goi O 18 trong tam cia hinh tit diém phing { A), Ao, As, Aas } vai P 1a giao digm hai duutng chéo Ay Ag va AAs thi trong tam T cla hinh ti gide phang la hinh vity cua diém P trong phép vit V(O,2), L0P. nghia la OT = 68 5.8 Cho hai duimg trdn (O;,R) va (O2.Ry) tiép xite ngodi nhau &digm C. Mot 6c vudng xCy quay xung quanh C; Cr va Cy ca (02) theo th ty 6 A va B, Tim guj tich hinh chigu vuong géc H ciia C tren AB. 5.9 Cho mot géc rOy va mot diém A nim trong géc dé, Hay dung mot dutng trdn di qua A va tiép xtic véi hai eanh cila g6e da cho. 5.10 Cho hai dudng trdn (vy) va (v2) dng tam O. Hay dymg mot day cung AD iia duting tron én cait dutmg tron nhd 6 B va C sao cho AD = 3BC. 5.11 Trong mat phang cho hai digm phan bigt A va H va mot duimg tron (0) di qua A. Hay dung mot tam giée ABC noi tigp (O) nhan HT lam truc tam, 5.12 Cho mot hinh vien phan Ay B cia duémg tron (O) [xdc dinh bai day cung AB va mot cung AB ciian6]. Hay dung hai dudng trdn (v1), (v2) tiép xtie ngoai nhau va cing ndi tip hinh vién phan da cho. Bién luan. B. Van dung phép dong dang vao viée giai cac bai toan sau day 5.13 Cho CD 1a duting cao ha xudng canh huyén AB ciia mot tam giée ABC wuong &C’, Ching minh rang céc tung tuyén AM va C/N ciia hai tam giée ACD va CBD wuong géc v6i nhau. 5.14 Cho hai hinh vuong eting hung OABC va OA'B'C! c6 chung dinh O. a) Chimg minh ring AA’, BB! va CC! déng quy; b) AA! vuong géc va bing CC’, b) Tim do lén cia gée gitta cfc tia AA! va BB', AA va CC! sitOAA!,OBB' va OCC' 1a ba tam gide cin cing dinh O, bing nhau ¥2 cing hudng. Chimg minh rang cdc c4p wong ing BC.BIC':CA,C' A’ ABC va A'B'C' giao nhau theo ba diém A,, B, vaC,, tao thinh AA,B,C,- AABC = AA'B'C'. AB "D 1a mot te gide 16i noi tigp mot dutmg trdn tam O. Phép quay xung quanh O bién né thinh ti giée A’B'C'D'. Ching minh ic dinh ciia mot cia hai ti gide d6 edt nhau ta hinh binh hanh, 5.17 Dung ra phia ngoai mot tam giée ABC bat ky ba tam gite BOM.CAN (BP sao cho ZMBC = ZCAN = 45°,ZBCM 0° 15°. Chimg minh rang tam gic MNP vuong can & 69 5.18 Mot hinh vudng ABCD cé dinh D e6 dinh va dinh A chuyén dong tren mot dutng (7) cho truée trong mat phing, khong di qua D. Tim quy dao chuyén dong cia hai dinh B,C’ cdn lai va ciia tam O hinh vudng ABCD trong cdc trudng hgp sau day a) (7) Bi mot dug thing a bo 5.19 Mot diém P chuyén dong trén nia dung tron dung kinh AB. Tim quy dao chuyén dong ciia hai diém M va N tren dutmg thang (8.1) sao cho PM=PN=PA. 4 mot dudng tn tam S, bin kinh R. 5.20 Cho hai dutmg tron (O;, i) } va (Oy, Ry) cat nhau & A va B. Hai dong tit I i dong tron déu trén (04) va (O2) ng mot huwrwéng, sau mot vong tré lai A cing mot Ic. 1. Chimg minh ring a) Tam giée AM; M, luon déng dang véi chinh né va dutng thing My Mp luén di qua B. b) Trong mat phang cé mot diém P duy nhat luén cécl 6 moi thai diém (Dé thi Todn quéc t& IMO, London 1979). 2. Tim qily dao chuyén dong clia cdc diém sau day a) Trung diém M cia doan thing My Mz; b) Tam C' dutmg tron (AM, My); c) Trong tam G, true tam H cia tam giée AMy My. |BC da cho, 6 dinh P cho truée XYZ cho truéc. 5.21 Dung tam giée MNP noiti¢p tam gis trén canh AB va déng dang v6i mot tam gi 5.22 Dumg m6t hinh binh han noi tiép mot hinh binh hinh ABCD cho tuée va déng dang véi mot hinh binh khdc MN PQ cho truée. (6 ‘anh A ABCD biétténg dé lénhai gécd6i dign. a,BC =b,CD=¢,DA=d. 5.23 Dung mot ti @ va do dai diah 3 (thudn) véi tam gi tich nh nha. déu BOA va CAB}. Goi ‘im ciia cde tam giée d6 va P 1A tung diém canh AB. 5.25 Dumg ra phia ngo: Ao va By lin lugt la Ching minh ring a) Hai tam giée A,B, P,B,A,P la vudng & P déng dang nghich véi nhau 70 b) Hai géc A, PB, va APB, c6 chung nhau duémg phan gise. 5.26 Mot diém P chuyén dong trén dutmg thang chita canh BC cia mot tam gitic ABC khong yuong da cho. Cée duimg thang di qua P vudng géc véi AC va AB theo thé ty eat ede dutmg thing AB va AC & M va N. Tim Quy tich cia diém Q, doi xing véi P qua trung diém cla MN. 5.27 Cho mot dudng thing p, mot diém A ten p va hai dudng tron (v1) va (v2). Dung mét tam gide ABC nhan p lam dung phan gidc trong cla géc A, cdc dinh B va C theo thif tw nam trén (v1) va (vz) va AB/AC = m/n (m van Ii hai s6 duong cho tru6e). © Wea <6i xing) 326, 527 dOi hi van dung tinh chat eta phép déng dang nghich (vi t= i toan 5.25, Chuong 3 Tra ldi va hu6ng dan giai bai tap §1 1.1 Goi M, va NV, Kin lugt 1a tip digm ciia a va b véi duimg tron (0); n6i kh di, ta c6 thé viet nhur sau dé dign 18 ky higu d6: (M,M,) = a,(N.No) = b. Ta xét hai trudng hgp: 1) Néu a||b thi diém M, ten a khong 6 inh tren b edn diém N, tren b khong c6 to anh tren a. 2) N&uaMb = C thi M, trén a ¢6 inh 1a C ben B, N, tren b c6 tgo dinh 1a C ten b; nhung My tren a, & 46 tiép tuyén this hai véi (O) song song véi b khong c6 anh tren b va MM, tren b, & 46 tigp tuyén tht hai vi (O) song song vi a khong c6 20 nh tren a, Nhu vay thi cd hai trudng hop a song song véi b va.a cat d, nh xa f: Mv N tira b khong phai [A mot song Anh. Ta di dén ket huan rang mud cho f = M ++ N tira +b 1A mot song nh thi mdi dung thang a vib déu phai bo di mot diém. Néu a song song véi b thi f 1& mot song nh tir a\{.M,} den b\{N,}. Con néu amb =C thi f 1 mét song anh tir a\ {My} dén BLN} Nhén xét. Néu ta bé sung vio mai dutmg thang a va b mot diém méi ma theo true gidc, ta goi 1a diém 6 v6 tan hay diém xa v6 tan, xéc dinh béi phuong ciia cdc dutg thing dé 1a anh xa f : M+ N tir a b lai wri thanh mot song nh (vi néu a || bthi M, tréna.c6 anh la diém Ng tren b va Mz trén b c6 tao inh I M, trén a; cling vay, néu ab =C thi My Noo trén VA Mao > Ni. 1.2 Néu a 1 b =O thi O tuong ting véi moi diém Mf cha mat phing. Néu a. Lb cin xét hai tudng hop. 1) aNd =O (c6 nhién a Jb), hay ching 16 ring MI" 1A truc tam ciia tam gidcOAB. Tir dé suy ra anh xq f: MM! ti’ + P 1a mot ton nh, (tie f 4 mot phép bign hinh ciia ‘P) khi va chi khi a Lb, 2) a|| thi diém MM" duge hoan toin xée dinh va duy nhat, C6 thé ching minh ring khi d6 f =‘D(A) la phép d6i xiing ¢6 truc A song song va céch 1 2 déu a vab. 1.3 1) Hay chimg minh ring digm M phai tim 1a giao digm thé hai ca dutng thang A va duimg ton (OPN) trong dé .N' doi xting voi N qua dung thang a. 2) MN =2QO (khong déi), trong dé Q 1a hinh chiéu (vudng géc) cita P trén A. i s6 khoding cach dn hai canh 1 1.4 1) Co thé sit dung quy tich nhing diém mi cla mot géc 14 mot hang s6 k [a mot tia nim trong géc dé (khi k fan gidc cla géc). Cing c6 thé xét cde diém A’, luot qua cdc canh BC.CA,AB cita tam gide ABC, 16i chimg minh AM,BM, va CM lin luot 1a phan gide eae g6e (C', ZC'BA' va ZA'CB!. Diém M" 1a tam dudng tron ngoai tiép tam 2) Chimng t6 ring khi M/= A thi A la bat ei diém nao cia duémg thing (BC). 3) Ching ta rang Ax! By! Nz! = M' khi va chi khi A’, B’, va C! khong thing hing réi ép dung dinh ly (céin va dl) vé duimg thing Simson, 4)T = P\To, trong d6 To = (BC) U (CA) U (AB) U(ABC) Dudng tron (ABC). 1.5 Sitdung tinh chat ciia phép ddi inh (bao ton kho’ng cach) néu trong dinh, nghia. 1.6 Sitdung tinh chat cia phép doi hinh néu trong He qua 1 ca Dinh ly 1, suy ra phép ddi hinh bio ton tong quan lién thude gitta dim va dudng thing, kéi hgp véi phuong php chimg minh phan chimg, 1.7 Goi M 1a mot diém bat ky ciia P va M' = f(.M), ta ching minh J M(¥M) bing phuong php phan chimg, Néu M'-2 M(¥M) th ing hing trén trung tryc f[./M'), mau thudn véi gid thigt (A, B,C thing hing). B.C ong, 1.8 Sirdung céc tinh chat cia phép di hinh: bao ton khodng each, bio toan ti sO cila hai dogn thang cing phuong, bao toan géc va cdc tinh chat dac trung ca trong tam, truc tam va tim céc dung tron noi, ngoai tigp ciia tam gic. 1.9 Sitdung tinh chait: Néu A! va B' 1a.inh ciia A va B trong mot phép tinh tién ‘T(@) thi A'B' = AB. Suy ra A'B' || AB néu AB I? va (4'B’) = (AB) néu AB|| v. 1.10 Tap hgp nhimg duimg thang bat bign trong phép d6i xing tam O 1 chim dutmg thing tam O. B 1.11 Nhimg duting thing bat bién trong phép d6i xting - truc A ky higu D(A) 12 nhing duting thing vudng géc véi truc déi xing A, con nhig dudng thing 6 phuong bat bien qua ‘D(A) bao gém nhiing dutng thang bat bién vung géc v6i A) va nhimg duémg thang song song v6i A. 1.12 C6 tat cd sd phép trong dé c6 ba ddi hinh thudn va ba ddi hinh nghich (461 xiing) Ba phép di hinh La Id. Q(O,27/3),Q(0,47/3). Ba phép di xing la D(t[BC]), D(t{C A), D(*[AB)), trong dé ¢/XY] 1a trung tryc cita [XY]. 1.13 Cé tt ci tim phép bao gm b6in di hin thusin va ben dd inh nghich; bao Id = Q(0,0),Q(0,/2),Q(0,n) = D(O),9(0,3=/2), trong 46 AChBDIutanisahuone, wk AC), D(BD), DAB), DBC). 1.14 Chi cé hai phép d@i hinh 1a Id va ‘D(O) bien hinh binh hanh ABCD thinh chinh né, trong dé O= AC BD 12 tam hinh binh han, 1.15 C6 tat cd bon phép: Id,'D(O) 1a hai phép doi xiing - truc qua trung trye cla mot cp canh déi dién. 1.16 Néu Ay L Ay =O thi ta 6 (v6i ky higu D(D,) =D,5 = 1,2) D,0D, =D, oD, = D(O)}; D,0.DO) Dro D(O)= DO) oD2=D § 2 2.1 Dinh ly nay duge suy ra tiy Dinh ly 3 vé su-xde dinh mot phép ding cu (bao g6m doi hinh va phan ddi hinh) phing. Cap diém tuong img thit ba C.C" chi c6 ¥ nghia dé x4c dinh phép doi hinh (dang cy) duge xac dinh bai hai tam gide bing nhau ABC va A’B'C (trong dé cic cap diém tuong img FA A,A'SB,B! va C,C) [a thugn hay nghich, tuy theo tam gige A’B'C" ching ‘ng hay khdc hung véi tam gife ABC. 2.2 C6 tat cd hai phép ddi hinh gm mot dai hinh thu va mot ddi hinh nghich bién tam gide thif nhat thinh tam giéc thit hai, trong dé A> A'.B > BIC Chote Avs A'Bi CC B!. Néu ABC [a di hinh thuan thi ABC 4 4’B/C' la doi hinh nghich, Néu ABC A'C’B! ta dds hinh, nghich, 2.3 C6 tat c su phép dang cu’ bao gém ba doi hinh va ba phép phan chiéu (ddi hinh nghich). Cic phép d6 duge x4c dinh bai cdc cap tam giéc déu bing nhau sau day: ABC,A’B'C'; ABC, B'C'A', ABC,C'A'B'; ABC, A'C'B' va ABC,C'B'A' 4 2.4 Vikhong c6 mot c’ip canh tuong ting no song song nén phép ddi hinh duy hat bign tam gide ABC thinh tam gide 4’ BYC' (hai tam gide nay nay bing nhau theo gid thigi) phai I& mot phép quay tam O ndo d6, Do dé ton tai O sao cho O= tA’) N¢[BB NCC. 2.5 Sit dung dang chinh tic cla phép doi hinh nghich trong mat phing. Céc diém A,,B,,C, thing hing én truc A cia phép d6i xiing - truot true AD(A, v). Tuy nhién, cing c6 thé chimg minh trye tigp tinh chat nay bing cach ké them dung phy. Ching han, truée hét thue hign T(7) v6i At bign ABC thinh 4’B"C" sau dé ching minh céc trung diém By va C, cia c4c doan thing B'B” va C'C” va A’ 1 ba diém thang hing. Tird6 suy ra digu phai chimg minh, 2.6 K¥ higu Aid = 1,2,3 [A cfc truc di xting va ‘Dj = D(A,). Xét tich f=D30Dy0D; (1). Tich Dy oD, 1 T(v) hoge Q(O,y) wy theo Ay song song hay giao v6i Ay tai O. Goi Ag [i truc thif tr dutge dumg sao cho DoD, =D20Ds. Tén tai D, theo cdc Dinh ly 4’ va 5’). Thay vao biéu thite (1) cia f thi duge f ='D,. Day [a digu phai ching minh, 2.7 DE ¥ ring Dinhly 8 ( 2.6 és tren) la digu kien dit dé f = DyoD.0D, 1a mot phép doi xiing = tryc ‘D,. Con trong bai todn ny ta phai thiét lap digu kign can va di dé ba dutng thing A, Ao, As cing thude mot chim dug thang (déng quy hoae song song). Déla f = D30D,0D, =D, hay 1a Dy oD, = D5 0D, (2). Céc tich & hai vé ciia (2) déu biéu thi mot phép ddi hinh D nao dé, nghia 1a hoac mot phép quay Q(O, y), hodic mot phép tinh tién T(2}) MAO =A, NAy= Ag Ag hoae # 1 A, (i= 1,2.3,4). Tir d6 suy ra dau higu nhin bidt can tim, 2.8 Sit dung Dinh ly 5', phan tich: Qx(O1,91) = D20.D; va Qx(On,) = Do ‘Do trong 46D; = D(A,) va Aa = (0102). Suy ra Q20@Q, = Dyo'D2. trong 6 (41,0) (mod 7) va Ao, Ay = $ va (Ai, As) = 4 ertya) thr. Cudi ciing dua dén vie xét hai tutng hop y+ y2 4 2ke vA Yy+! 2.9 Sirdungeac Dinh ly 4’ va5'r6iphan tich Q(O, 9) va T(3) thnh tich ciiahai D(Ai) =D. Cudicing thu duge f = (i )oQ(O,¥) = DyoD: = Qw, 0) trong d6 O © A, sao cho (AryA2) = g/2 (mod); 0 € Ar Lv va As suy ra tir Ay bai TB) vaw QO, gor) ciing am tuong tu nh f. Ky higu Ai =DgoD, = Q(w!,e). tong ds D, = D(A), 240 Tir gid thiét ta c6 AABC = AA'B'C’. Sau d6 sit dung Dinh ly 3. Tén tai a Ars Al Bes BICC. Sirdun; hiét hai tr joi nén D nam khée ph i DI khie véi B! d6i véi ABIA'D ABIC'D! va f cing 5 bign D+ D! va do dé A'B'C'D! = hoje = ABCD tu theo hai tt gide nay cling huting hay nguge hudng. 2.1 Tra lav. C6 hai phép dng cy bign ABCD thinh A'BIC'D', tone 46 mot doi hinh va mot phan chiéu, Phép doi hinh 1a phép quay Q(A,). trong ds ‘AB, AB’), (mod 2r) hode ¢ = (AB,AD"), (mod 2n) tuy theo hai hinh vung cling hung hay khéc hung. Phép hen chiéu Ia phép d6i xing = trye ‘D(A), trong dé A =¢{CO'] 1a tung tryc cia [CC"). 2.12 Sir dung he thite vécto bigu thi HT 8 tryc tam ciia tam gid hay chimg 1 ring bén dogn thing A,H, c6 cing trung diém O' déi xting vi tam O cia dung trdn ngoai tigp Aj A»AaAs qua trong tam G cia tit dim (Ay, Aa, Aas Aa}. § 3 A 3.1 Goi AD.BE va CF 1a cfc trung tuyén cia tam gide ABC. Thue hign vige tinh tién BE theo # = BD, dua BE dén DP thi F tr thinh trung dim chung cha C/A va PF. Tird6 suy ra AD+ BE+CF = 0. Vay 3T(AD,BE,CF) = AADP 3.2 Tra lai. AH = V7 =P Chimg minh truc tam H cia tam giée AEF 1a dinh thit wu cia hinh binh: hinh ECFH. Sau 46 thue hign tinh tign theo HF réi suy ra AH = GF, trong d6 G 1a dinh thé wy ciia hinh binh hinh AH FG. duimg ton (A,r) trong 46 O 1a tam cla dutmg ron (O,r) chuyén dong di qua A cé hai tigp tuyén 1,2) song song vai A ma cdc tip diém T; d6i ximg nhau qua O sao cho T\T, 1 A. Td lat. {Ts} va {12} 1h hai dung tron (Kar) va (Kar) duge suy ra {O} bai hai phép tinh tién theo AKy = 7 AR = —F,, wong d6 [K Ky] 18 dudng kinh wuong géc véi A cla (A,r tir dudmg tron (A,r) HK, twong 46 h cla dutmg tron (CDE). K =(MD)N(NE) ACK la mot ding 16 Cudi cing, thiét lap he thite 25Q = BC + AH. Suy ra SQ = OC, trong 6 O la tam dug tron ngoai ti¢p tam giée ABC. 3.5 Gid sit d(B, CD) < d(A,CD), trong 46 d( X,Y Z) laky higu khoang ciich tirdiém X dén dudmg thing YZ. Thue bi =NB)thi No Bva M+ D sao cho MN BD lamothinh binhhanh vadod6 DB ||= MN 3 as = Dat MN = @ va goi / 1A véeto chi phuong cla dutng thing (CD) chita duing kinh [CD] cita (O). Suy ra BD = =6 va MD || (BN) = (BP), do d6 ZAMD = ZAPB =. Diém M duge hoan todn xéc dinh, M1 mot trong cc giao diém ca [CD] va cung (7) chifa géc dinh huting y dumg trén doan AD, trong 46 BD = =F. Bai ton ¢6 hai, mot hose khong c6 loi gidi tuy theo (+) cat, tip xtc hay khong cat [CD]. én A || d va cat (Q) va (O’) theo hai day cung tuong tng 1a MN va M'N' sao cho MN + M'N' = € cho trude, Kéo dai MN’ v6 phia N’ ly diém My sao cho MIM, = 2 ri dat MAN = € va goi 7 Ia vécto chi phuuong cia d. Thue hign (7) = (0) + (1) c6 tam QO; xée dinh bai OO, = TU. Sau dé thuc VM, bign (O') thinh (w). ‘The thi A cat (w) theo day cung i+ MN = MIN . trong d6 JV; [a giao diém thi ia A va (Oy). Suy ra w = O'w L t{OOjJ trong dé #00, Ia trung trye cia doan [00;]. Ket luan: Cat tuyén A phai tim li dutng thing di qua mot giao diém cia hai dutmg trdn (Oy, R)s (w, R’) va song song véi d, trong d6 R va R’ Kin luot 1a ban kinh ciia (O) va (O’) 4 cho. Bin ludn: Bai todn c6 nghigm (hai hoje mot) khi va chi khi 3.6 Gia sit da dung duge cat 1 n tinh tign theo é |R-R<5 Rhoge P nim khic phia v6i (O) d6i v6i a, trong d6 R la ban kinh ca (O), d; = d(O,a) va dy = d(P,a). "(| BC), PQU CA) va RS|| AB) 3.12 Badudng thing efit M song song véi BC,C'A va AB cia dutmg tron (1) noi tigp tam gide ABC. 'S thu duce khong nhiing ngoai tigp dung tron (7) ma con nhan 7 fam tam doi ximg. nal 3.13 Thue hign phép ddi ximg - tam D(P) : M4 NS CD va cung chifa géc dinh hung x = (mod 27) dung tren doan thing [BIA], trong dé B' doi xting v6i B qua P. Suy ra $= [AM)N(O). Bai ton ludn c6 Idi gidi. c 3.14 Hay dé ¥ dén tinh chat sau: Du’ng n6i tam Ow ciia hai dutmg tron (O) (w) 18 tryc d6i xting chung cita hai duémg tron 46. Néu chéing cit nhau thi dutmg n6i tam hai dutmg tron 1A trung truc cia day cung chung cia ching, 3.15 Hinh 1. gém dung tron (O) va hai dug tron bang nhau (O;); (2) cing tigp xiic trong véi (O) 06 tryc doi ximg 1a trung true A cla O1,0> tring véi duimg kinh ciia (O) vuong géc véi Ay Aa ciing 12 #[A;, Ap). D(A) bién M+ M',By + Bi,By > B! MM' || BB; || BoB vi ‘Mf thang hang va As, B}, M' cing thing hang. Mat khéc, phép vi ty V(Ay, 2)/ Re) bién By BS thinh MAM" || By Bs vi do dé suy ra (B,B3) = (B,B%), nghia 1a By BS || MM’. Suy ra By By || AvAo. Chimg minh nay, ngoai phép déi xing - truc cdn sit dung tinh chat ciia phép vi ty. B Chi thich. C6 thé ching minh true tip tir AA\O:B, ~ AA\ON AA,02My + AA,OM, sau d6 dp dung dinh ly Talét chimg minh 74 M24 suy ra By Bs || Av 3.16 Phép D(AB) CD +> C'D' LCD =M € AB. Suy ra MC! khdc, vi ZC/CD = 45° nén C'D khong di Tra lai. MC? + MD? = 2R? (R= 1AB), 3.17 {M 318 BD) NA} Ia dutmg trn ngoai tigp tam giée can ABC da cho. fi sithai guong thing gi vag» duge dat sao cho hai mép guong cia ching sat nhau tgo thinh canh c cla mot géc nhi dign Ze( 91, 92) 06 g6 1h gc (nhon) zOy: Ox c,Oy Lc. Bé tinh do lén cia géc sit dung dinh lugt quang hoe: géc phin xq bing géc t6i Tra lai. ZeOy = 45° ké tir A v6i dudng tron tam B' tip xic véi ry, trong d6 B' Adi xing v6i B qua cy. 3.19 Tra ldi. Diém P cén tim la giao diém cia ry va mot tip tuys 3.20 1) Goi D' = D(D), trong 46 A = ¢[AC] 1a trung true cia AC. Khi d6 vi ABCD 1a mot ta giéc 16i, ta thu duge bat ding thite cdn tim. 2) Dau dang thiic & (*) dat duge khi va chi khi déng thdi ta phai c6 s(BAD’) = $A4B.AD! va s(B'D'C) = $BC.CD! ZBCD! =90°,t hai dudng chéo AC, BD wudng géc. D 3.21 Trd lai. Céc phép ddi hinh bin (O,) thinh (02) g6mT(v? = 0,02), D(O), trong 46 O Ia trung dim ciia [0,02] va v6 s6 phép quay c6 tam w nam tren A = ¢[0; 02], Cac phép phin chigu gdm ‘D(A) va v6 s6 phép doi xing = trugt c6 true d 1a mot dung thang bat ky di qua O. 3.22 Trude hét, chimg minh tinh chat sau day eiia tam quay: Trong mot phép quay khong phai 1A mot phép doi ximg - tam thi tam quay, hai diém wong ting va giao diém cia hai dutmg thang twong ding phat xust tirhai diém d6 bon diém ciing thude mot dudng tron. (Sir dung géc dinh hung (mod 7) ai dung thang). Hode chig minh tinh chat: Trong mot phép doi hinh ), g6c gitta hai duémg thing tuong ing c6 46 ld (dai s6) khong adi. Goi y = (a,b), (mod 7) va O 1a tim cia phép quay Q(O,g) bign doan thing My(ty)M(ty) thanh dodn thing Ma(t,)Ma(ta), trong 6 tyst2 1X hai thoi diém nao dé (¢) > ta). The thi phép quay Q(O, 9) s@ bién My(¢) thinh MA(t) & moi thoi diém t. 79 3.23 Gia sir (Oa, R) (Oa, R) = (P,Q) va goi (POs, PO? sit mot cat tuyén A bat ky di qua Q cit lai (O2) & M, minh APM, My» APO\O». 6 (mod 2n). Gia 1,2; hay ching 3.24 Goi O 1a tam ciia hinh yudng AyAyAaAg gid sit 4 duoe dinh hung Xét phép quay Q(O,/2) bign hinh vuong thanh chinh né, A; > Ay be thuin tign, ta ky higu lai c,y,2,¢ theo thé ty ddi thanh 21, 02523524 (vs xem 1A x4). The thi Q(O,x/ AyeaiAiti > Aiy teas Ady raphépquay Q(O,—m/2) :AyP + Ay-aty-1 vdi quy w6c 9 ars. Tre lat. Diém déngquy cba Ax; ladiém Q duge xéc dinh boi OQ = OP va (OP,0Q) =—7/2, (mod 7) dugesuy ratit P bi phép quay Q(O,—7/2). 3.25 Xét phép quay Q(C,+60°) : Br M', Nv A vado dé Di B. Day la digu phai chimg minh, 3.26 Sir dung tich ciia hai phép quay (dinh ly vé tich cia hai phép quay da duge hig minh trong bai tp 2.8). Gid sirtam gide ABC cé hung duong. Ta ky higu Qi = Q(B, 7, 27/3) roi xét tich f =Q2 Qu, chimg té ring f= Q(A,,—7/3) tir dé suy ra tam giée A,B,C, Ia déu 3.27 Hay sit dung phép quay g6c 60° theo ¥ tuéng néu trong Idi gidi cha Thi dy 10; §3, 3.5. (tig dung phép quay), Chang han, ta thyc ign phép quay Q(B,60°) 0Q(B,60" M d Tidu. Xé ANAM 66 1 MB chinh 1a tam giée T(MA, MB,MO) T(MA,MB,MC) = AN. thitc) sau day (NB, NA) =(MB,MC)=(NM,NA) + (AB,AC), VM EP Tir dé suy ra (NM, A) = 90° hay 1a (MC, MB) =30°, (mod 180°) Tré lai. {M} wrong mit phing sao cho tam gide T 6 ba canh bing MA, MB vi MC latam giée vuong gém ba dutng tron bing nhau c6 bn kinh bing canh cia tam gide déu ABC vadoi mt tigp xiie nhau 6 cdc dinh cla ABC. Tam eiia ba duimg tron quy tich nay 1a cae diém A,B, vi C, li cdc diém A, By va C, li cdc dinh ciia ba tam giéc déu doi xting véi tam ABC qua cac canh cia né. 80 g 4 4.1 Suy ra dinh ly 21 vé sy xée dinh mot phép 2 phing (xem chi thich 6 cudi chimg minh dinh ly 22, § 4.42). Céch chimg minh twong ty eéch gid 1a mot phép tinh tién vi k= B'C'/BC ou 1) va bat ext dudng thing no di qua hai diém twong img déu di qua tam vi tu. Chui thich. Cing c6 thé chimg minh truc tigp bing céch sit dung dinh ly Tale. 4.3 Thue chat day 18 mét bai todn dung hinh. Gi [BC] sao cho MN || AB || CD va [DC (AD) (BC), hay chimg ta rang OM Suy 1a 0D (ON =OM/OA= y bin Ds MM + A.C 4 N,N v4 B tte t8 V(0,\/98) bien binh thang DCN M thinh hinh thang MNBA. sire6 digm M € [AD], Ne |» [IMNBA. Goi O OA.OD va MIN’ = AB. do 6, phép vi ty duomg V(O, f 4.4 Suy ra tit vige tam gidée PA; By déng dang nghich véi tam giée PByAa va cp tam gidc déng dang nghich nay c6 cap dudng trung tuyén tuong dng 1a AoBr va Bp Ay cing v6i clip dinh tuong tng Ar, Bre 4.5 Truéc hét, hay thiét lap dau higu (diéu kién di) nhan biét hai hinh chirhat 1 ddéng dang (suy tirdaiu higu nhan biét hai tam giée vudng déng dang). D6 [a 1/86 hai Kich thuée cia hai binh chi nhat, bing nhau. Gia sirbinh chit nhat MNPQ ngoaiitigp tt gide (I6i) ABCD 06 AC 1. BD; hay ching t6 ring MN/MQ= BD/AC (néu AE MN, BE NP.CE PQ va DE QM). 4.6 Sit dung géc dinh hung cia hai dutmg thing (mod 7) chimg minh ec he thc (4’B', A'C") = (HB', HC") = —(AB, AC) va hai he thite twong tr suy ra tam gide A’B'C! déng dang nghich véi tam giée ABC. 4.7 Tré lei. C6 tat cA siu phép déng dang (thugn va nghich) nhut phép Z thuan va ba phép Z nghich bién tam giéc déu thi nhat gic d&u thit hai. Hay chi ra dit sdu cp tam gide déng dang (goi ten) c thudn va nghich (lién hé véi bai 2.3). 4,8 Ménh dé ddi hoi chimg minh dala mot phn noi dung cita chimg minh Dinh ly 22 vé digm bat dong va dang chinh tac ciia mot phép déng dang thusi 81 4.9 Hay ching minh ring néu O 1a tam cita phép déng dang thudn (vi ty quay) 2,(0,¢,k) bién AB thinh A'B! thi AOAA' ~ AOBB' va ciing vay AOA'B! o AOAB. Tit dé suy ra O ciing 1a tam cha mot phép Za thuan khéc bign AA! thinh BB’. 4.10 Lai gidi 1. Sit dung céc két qui néu ra trong : trén, Diém déng quy O cita bon dutng tron (AA'C’), (BBC), (A Bo) (4'B'C) chinh la tam chungetiahaiphép déng dang Z, bign (01) = (ABC) thinh (O2) = (A’B'C); va Za : (Os) = (AA'C) 4 (Os) = (BB'C). Loi gid 2. Sic dung dinh ly vé dung thing Simson 4.11 Dat (PO, PO") = yp (mod 2n), k= R'/R. Xét phép vi tu quay 2(Pg,k) bien (O,R) thanh (O',R'), tong 46 O + O'. Gidsit Z: M © (0) 4 M' € (0'). Hay ching t6 ring Z : (OP,OM) + (O'P,O'M’). Tirds suy ra(M,M’)3Q 4.12 1) Ky higu (01) = (APEF), (2) = (PBCD) va (0:) (02) = {P,Q} Sirdung ket qua bai tap 4.11 Grén hay chiig td rang phép vi tw quay tam P da bign (O») thinh (Q,) thi cing bign hinh vuong PBCD thanh hin wong PEF A, tong dé Bs B,C + FD A. 2) Diém c6 dinh S 14 tam hinh vuong dyng tren [AB] nhung nam khde phia v6i hai hinh wong duge xét d6i véi (AB). Con {Q} 18 nita dutmg tron dutmg kinh [AB] nam khéc phia véi S ddi v6i (AB). 4.13 Ching t6 ring c6 mot phép vi ty ddi xing tam O bién A A.B BUCS CDAD’. 4.14 1) Dé thay f : My M' tir PP 1a mot song Anh tix P vao chinh n6. 2) f limo phép déng dang nghich Z ciia mat phing; Z = 2(C,A,k) tam amb, ruc A 1a phan gide Cp cia g6c tao bai a 18. géc nhon tao bai a vad; f chinh 1A mot phép vi tur doi xing. §5 A 5.1 DEY ring A) = D(P)=V(A,)s trong dé V = 5.2 Goi G Ia trong tam cila ti diém (A, A2A3Aq} chimg t6 ring B, duge suy ra tit Aj, (i = 1,2,3,4) trong phép vi wr V(G,—1/3). Tam O; ela dutmg, tron (B, ByBsB,) duge xdc dinh bai GO, = -4GO. Ry /(P, 2), wv. 82 5.3 Dat AD = BC =d (khong d6i) suy 1a {C} = (B,d) va do 46, {O} due s suy ra tir {C}} nhi he the AC =240, hay AO = 3 5.4 Dat 1B =a,AC = b; suy ra BP: BN =k tim duge {P}. 5.5 Goi I va O Kin lugt 12 tam cc dutmg trdn noi va ngoai tiép tam gic ABC. Chimng té ring ABC va A,B,C, 1d hai tam gide vi tu va D, 1A tam dudng tron ngoai tip tam gide A,BoC. 5.6 Goi K Wa tigp diém cia (J) va dubng tron (ABC), H 1a trung WAT Ra trung diém MIN. Xét phép vi tw VA, AADE wong d6 H+ K Bat i 5.7 Hay chia migng phing déng chat c6 dang m6t nita tt giéc 16i (hai chiéu) [A1AoAsAq] thinh hai mién tam giée bang mot dung chéo nao 4, Ay Ay ‘Sau dé dua vao trong tim cia hai tam gic thanh phén (ma ta da h tim) thi xdc dinh duge trong tam cila toan hinh T = [By Ba) B2 Ba] »), ai trating hop: Ry # Ry va Ry = Ro. Néu Ry Ro goi I 1a tam i chia (Oj; R,)oi = 1,2 va D = D(C), Xét phép vi tu VL), k= R,/R, ching minh [CA] + DB); tir dé suy ra (AB)°T va do dé, "HI =90°. ‘Trai li. Néu Ry Z Ro thi {H} fa cung tron HyCHa cha dung tron dung kinh [7C] c6 hai du mat H,, (i = 1,2) nam trén hai tigp tuyén chung ngoai cia (O;) va (Oo). Néu Ry = Ro = R thi {7} 1 doan thang [Ff Ho] ciia dug trung tryc ¢[0; 02] cha doan néi tim hai dung tron va A, nim tren cae ti€p tuyén chung ngi hai dutng tron (O;, R). 5.9 Tam thdi khong di hdi digu kign dudng trdn phai di qua dug trdn (w) can dumg vi tr v6i mot dung trdn (7) bat ky tip xtic vi hai canh cila géc rOy da cho. Suy ra diém A 1a hinh vi ty hai giao digm My vA My cita dung thing OA v6i dutmg tron (7) 46. ‘Trai lai. Bai todin 06 hai nghiém hinh nhan ti tir boi phép vi te V(O, ky = OA/OM); i= 1,2 5.10 Theo gi hie, néu day cung AD cba dung tron Ion (01) d@ dung doe th AB =BC =CD : 4AC. Phép vi tw V(A, 3) bien C B. Suy ra B mo (2) va (vy) nhan duge bai phép vi tu néi trén, véi hai, mot, hoac khong Idi gidi Ry < Ry S3Re, wong d6 RF; | i méi diém A duge chon trén vy; tuy theo in kinh cita (2;)yi = 1,2. 83 5.1 Goi A,, la trung didi Dy,(H) vado a6, A, cia BC va A = Do( A). Hay ching td ring A’ = 5.12 Goi C 18 trung diém cia cung bit véi cung Ay B cita hinh vien phan da cho; D,,E, kin luot 1a tig diém iia (v),(i = 1,2) v6i day AB va véi cung ‘AyB. Hay sitdung tinh chat ca phép vi tw, chimg minh (D,E,) di qua C. r6i ching minh C nim trén truc dang phuong cita (v1) va (v2). ‘Trai li. Bai todn 66 vO s6 nghiém. Quy tich tigp diém cita (0) va (v2) FA cung tron AB cita dutmg tron (1') tim C, bin kinh p= CA= CB nim trong hinh vien phan da cho, B 5.13 Xét phép vit quay Z(D,90°,k), trong 46 k = AC/CB, bign BD thinh CD, C thinh A va do 46, bign CN thinh AM. i ti’p hai hinh vudng dicho va P a gi i ciia hai dutg trond6, Théthia) AA'MBBINCC! = b) CC" nhan duge tir AA’ béi phép quay Q(O, 90°). 2) Phép déng dang 2(O,45°,k) trong 46 k= V2 bién Ad! thinh BB. 5.15 Tam gide A'B'C! duge yy ra tit tam gife ABC bai phép quay géc (0A,04" (0B, 0B") ‘OC,OC") xung quanh tam O. Goi Ay A Ta cdc hinh chieu cia O lan luot wen BC vi BIC" hay ching ring ‘A, = BC BIC" nhin duge tit Ay bai phép vi tu - quay 2(0 Suy ra, AABC AA\BiCy > AABC = AA'BIC, 3) comer)» 5.16 Goi B,F,G, H lacéc inh chigu cia tam O Kin lugt tren cdc canh AB, BC, CD, DA cia tt giée ABCD noi tigp trong (O); cling vay, E', F',G!,H! cc hinh chiéu cia O trén cdc canh tuong tng cia A’BIC'D’. Hay ching t6 ring MN PQ nhan duge tirhinh binh hanh E/FGH bai phép vi ty - quay tam O, géc p/2, tY s6 k = 1/cos(y/2)- 5117 Sit dung tich ciia hai phép déng dang thuan, Xét tich Zo Z. trong dé 2, = 2(B,A5°.ky) va Z) = Z(C,45°, ky) trong dé ky = BC/BM = AC/AN = 1/k (vitam gide CAN déng dang nghich véi tam gide CBM). Hay chting t6 rang Zy 0 2; 18 mot phép ddi hinh c6 mot diém bat dong P duy nhat va géc ddi hinh y = 90°. Tirdé suy ra PM vuong géc va bing PN. 5.18 Gia sit hinh vuong ABCD cé huéng thuan. Thé thi, Z(D, 3, v2): A> B:Q(D,3);A> C vaV(D,8): BO. ‘Tré loi. 1) Goi H La hinh chigu cia D tréna r6i dung hinh vuong DH H'K 6 huéng thuan. Thé thi ta duoc { B} la dutmg thang b vudng géc voi DH" GH {C} L DK=K;{0} = (HK). 84 2) Dumg hinh vudng DSS,S» 6 huéng thudn (cing hutng voi ABCD); goi w Ia trung diém cia DS,. Ching 6 judg tron (S:RV2), quy tich ciia C 1a dutng trdn (Sy, RV2) va qui tich ciia O la dutmg tron (w, RZ). 5.19 Goi tén cic digm M va N sao cho tam giée APN’ nguoc hudng véi tam gidc APM duge gia sir 1a c6 hung thuan, nghia 1 va AM = AN = APV2 (AP, AN) = -(AP, AND) a Tré li. Dung tam gidce AB, By vudng can 6 A, nhgn AB 1a dung caoha xudng canh huyén By Ba v i can 6 A. Chimg t6 ring {M1} 1a nita dutmg tron (ABB,) dudmg kinh AB) {N} 18 nita duémg tron (ABY By) dutmg kinh AB» (c6 tam wy i trung digm cita nita dutmg trdn da cho), trong 46 Bj doi xing véi By qua trung digm O cia [AB], déng thai BY =D,,,(B) va cing Ia dinh théttu cla bin wuong ABB BI. 5.20 1) Ching minh (0) A,0,.M]y) = (024,02), Vt, tit dé suy ra (My Mz) 3 B va tam giée AM; My déng dang véi tam giéc AO,O>. b) Trung truc #[M, Mb] 5 P Ia trung diém cia doan thang B, Bp, trong dé By = Do( B)s(i = 1,2) 2) a) {M} [a dutmg tron (O) dudng kinh BP cé tam Ia trung diém O 0,02; cin uu ¥ them ring {7} ciing di qua B vi cé (MA,MB) (khong Ai) do tam giée A.M,.M cling luon déng dang véi chinhn6. b) Quy tich cla C' 1a dutmg tron ngoai tigp tam giée AO; Oo. c) G duge xée dinh oe nS ne béi AG = 3AM; H duge xée dinh boi CH =3CG. Tra lai. {Gi} la dude tron (Go,P) = Goa), trong dé G, la trong tam cia tam giée AO\Oa; {H} fa dutng tron (H,,P2 = HA), trong 46 H, 1a trye tam ciia tam gic AO\Oo, 5.21 Phan tich. Gia sit ring tam gide MNP a dng duce, nghiia aM € BC,N €CAsao cho (PM, PN) = vak =ZY/ZX, Suyra, N= (CA)na’, Fhaaee han duge tira = (BC) bai phép vi tu - quay Z(P,7,k)- Bién lu@n, Bai ton néi chung c6 mot nghigm hinh duy nbat, tir wudng hop a’ || CA hoae a! = (CA) thi sO nghigm bang 0 hod oo. 5.22 Néu hinh binh hanh /'/P'Q' noi tigp hinh binh hanh ABCD thi ede tam O! v2.0 cita ching triing nhau (Bai tap 3.8). Nhu vay bai todin duge quy vé bai todn 5.21 6 ten, D6 1d bai todn: Dung tam gide OM'N' noi ti¢p tam ABC cé dinh O 18 trung diém canh CA va déng dang véi tam gide cho truée, trong dé MN 1a mot canh va w i tam cia binh hanh NPQ cho true. 85 5.23 Trude het, dé ¥ rang néu ZA+ 2 dung bai toén chinh 1 thé dy 11 tong 5.2. § 5. Nhu v: mot dang khai quat thi du d6, Tuy nhién, 101 gidi ciia bai todn lai ho’n todn tuong ty véi Idi gidi trong thé du d6. Trong bai vin quy vé dung ADCE % ADAB, chi khac la B ¢ (BC) vi ZBCE= BCD+2ZDCE LO+LA=O 40 5.24 Hay ching td ring bén dudng tron 14(AYZ),v(BZX),v(CXY) (XYZ) cé6 cing bin kinh, Sau d6, sir dung phép chigu vuong gée, chigu YZ len BC, ching minh ¥, Két ludn.nin S(AXY Z) = 1S(ABC) dat khi va chi khi 48 = tuong duong vi AXY Z = A,B,C, Tra loi. ‘Tam gide cn tim 18 tam giée trung binh A,B,C, cia tam gide ABC. 5.25 1) Ching minh céc tam gide A,ByP va B,AyP i nhiing nita cita hai tam gidc déu, vung & P, 06 ZA, 60° va ZB, = ZA, = 300 (sirdung dinh ly vé tich hai phép quay, bai 2.8). 2) Tirviec tam gic A,B,P déng dang nghich véi tam giée B,AyP suy ra cde doan [4,Ai),[ Bo, Pi] dutge nhin tir P duéi nhing géc bing nhau 1 5.26 Ching minh P 1a tryc tam ciia tam gide AM va do d6, Q la diém xuyén = d6i véi diém A trén dutng tron (O) ngoai tiép tam MN. Tra loi. Quy tich cia Q Ia dutng thang q, nhan duge tit duimg thing BC) béi phép vi tu- d6i xting 2(A, A = Ap. k), trong d6 Ap 1a phan gide géc ZBAC va k aig ty theo ZBAC 16n hon hay nhd hon 90°. Chui thich. Bai toan ndy chinh 1a dang dio cita Thi du 13 (VMO, 3/2002). 86 Hinh 52 Loi gidi. Gia sit rang tam giée ABC da dumg duge, thod man cic di hoi cita bai todn, The thi diém C duge suy ra turdiém B bai phép déng dang nghich (vj tu - d6i xing) 2(A,p,k = 2) tam A, truc p vaty s6 k= n/m Do dé diém C cain tim 1 mot diém chiing ca hai dutmg tron, dé 1a dung trdn (va) va dung tron (v}) nhin duge tir duémg tron (v1) boi phép déng dang nghich Z(4,p,n/m) n6i trén (Hinh 52). Bai tosin c6 thé c6 hai, mot hod vO nghiem tuy theo (v) clit, tigp xtic hoc kh6ng cat (vy). Chai thich. D@ thay rang bai t vo nghigm khi va chi khi dung trdn (vy) nim ngoai géc Zt, At, anh ella géc t Aty trong phép doi xing trucD(Ap) trong d6 At; 12 hai tigp tuyén ké tir A dén (v1), Day 2 mot thé du ing dung phép déng dang nghich (vi tu - doi xtng) vao gidi tosn dung hinh, xem 1A mot thi du bd sung vao 5.3, § 5 (Thi du 1.4). in Chuong 4 Bai tap b6é sung va On tap tu gia § 1 Bé tic géc dinh huéng trong mat phang 161 a) Gia sir A/ 1a motdiém nim trong mat phing ciia mot g6e rOy di Goi _X va ¥ Kin lugt | d6i ximg ciia M qua cée canh Ox Oy. Ching 16 ring 46 16n cita g6c ZrOy khong phu thude va vi tr ciia diém M (M40). b) Cho M li mot diém bat ky nim trong mat phang cia mot tam gidée ABC dicho. Goi A’ B.C" lin lugt la céc diém dei xing cba M qua cdc dutng thing BC,C'A va AB. Chimg minh rang cdc dutng phan giée Ar, By va C= ciia cdc g6c ZB'AC’,ZC'BA' va ZA'C'B dong quy hoac song song. Tim quy tich nhiing diém M trong mat phang dé Ax, By va Cz song song véi nhau. ho. 1.2 a) Chimg minh ring géc cia cde duimg thang chifa hai day cung AB va CD cita mot duimg trn (O) duge cho bai cong thie (AB,CD 204,00) + (OB,OD)} +kx b) Goi a 1a 86 do dai s6 cla cung AC nam trong gée ZADC va 3 1a 86 ia cung BD nim trong gée ZBAD, ching minh ring (AB,CD) = (AB, AD) +(DA,DC (a +8) +! 1.3 Gi sithai duing tron (ABC) va (ABD) trye giao véi nhau. Chimg ta rang hai dudng tron (ACD) va (BCD) ciing vay. 1.4 Goi B' la diém d6i xing cila B qua canh C'A va C' la diém doi xing cia C qua canh AB ciia mot tam giée ABC. Tinh B'C' theo ZA=avaOA AB=c 87 88 Bi vAC" theo thit ur dim d6i xing cita cdc dinh A,B vac canh BC,CA va AB. Tim diéu kién céin va dit vé dang cla ata |BC dé A’B'C' 1a mot tam gidic déu (Dé thi HSG toan quée, VMO 3/1994) 1.6 Gia sit DE, F 1a ba diém bat ky duge lay Kin Iuot trén ede dudng thing a AB cla mot tam gide ABC. ron (AEF), (BF) va (CDE) déng quy & im M, goi li diém Mi-ken (B6 dé Miquel). b) Tinh g6c (DE, DF) theo cdc g6c (AB, AC) va (MB, MC); ¢) Tim quj tich eiia diém M khi D, #2 va F thay d6i nhung luon thing hang 1.7 Cho mot tam giéc ABC. Dung ba dutng thang Ar, By (AB, Ax) = (CB, C2) va (AC, Ar) = (BC, By) Cy sao cho Ching minh ring Av, By va Cz déng quy & mot diém D tren dutdng tron (ABC). 1.8 Chimg minh dinh ly: Cac dutmg tron ngoai tigp bén tam canh elia mot tf gic (ti canh) to’n phiin déng quy & mot diém w m: hinh chigu ciia n6 trén cc canh li bon dim thing hing, hinh chiéu cia mot diém w nm tren mot dudmg tn noi ti¢p duting trdn d6 18 cde dinh cia mot 1.9 Ching t6 rang trén sdu canh cia mot ut tit gide (Wi canh) toan phéin, 1.10 Dinh ly Miquel vé sdu dutng tron: Bon duémg tn vy, v2.02 va vy sp dat trong mat phang sao cho v Mv = {4,4"},02M va = {B, Bsa 104 = {C,C'} vaws v1 = {D,D'}. The thi cde dim A’, B'.C'D! dong vien (hoe thing hing) khi va chi khi cdc diém A, B,C, D déng vien (hoac thing hang). 1.11 Cho mot duting trdn (O) va hai digm A,B e6 dinh tren dutmg tron dé sao cho kh6ng 12 hai digm xuyén tam - doi, mot dudng kinh XY thay ddi ciia (0), Tim qu tich giao diém P cia (AX) va (BY), (Dé thi Olympic Toan clia My, USOMO, 1976). 1.12 Trong mat phang cho tam giée ABC noi tigp dudmg won (0). Xéc din mot diém P khong thude dung tron (O) dé cae dutmg thing PA,PB.PC cit lai (O) in lugt & A’,B',C" sao cho tam gide A’B'C! vuong cin day BIC" (Dé thi HSG toan quéc, VMO 3/1999, bing B). 89 § 2 Dai cuong vé cdc phép bién hinh va cdc phép doi hinh phang 2.1 Trong mat phang cho ba diémt A, B, P phan biét, thing hing va mot dutng thang A vudng géc véi (AB) & D. Mot diém C chuyén dong tren A. Goi A’ vi BL hinh chigu cia A va B lin luot tren BC va CA, AA! eit BB! &diém H tren A. a) Hoi anh xa f : C+ H tit A+ A 6 phai 1a mot phép bién hinh cla A khong? b) Goi Af 1a hinh chigu ciia C’ tren dutng thing (PH), Héi anh xa hi: Hey M tit duting thang A len (7) = {M1} c6 phai 18 mot song nh khong [ bign A thanh y= h(A)]? 2.2 Trong mat phang cho tf giéc 16i ABCD noi tigp mot dutmg tron (O). Mot digm P chuyén dong tren dutmg tron (O). Cac dutmg thing DP va CP cat duimg thing A di qua A va B theo tha ty 6 M a) Ching minh ring néu bé sung vio dutmg thing A mot dim méi theo tryc giéc goi IA diém v6 tan hay dim xa vo tan ciia A thi Anh xq fz M+ N tir A vao chinh né 1 mot phép bién hinh cita dung thang A. b) Ching minh ring khi diém P chuyén dong tren dutmg tron (O) thi ty kép (AB/MN) c6 mot gi tri khong d dinh. trén , ching minh ring khi P chuyén dong tren (O) p (khong déi). XC theo thi hinh chigu tren AB cia mot diém A bait ky trén mat phang va G Ia trong tim cia tam gide A'BIC’. Chimg minh ring anh xq f : M ++ G tit P+ P la mot phép bien hinh ciia mat dn. Goi O la ong tam cia tam gide ABC, trung diém cia OM), ching minh G 1a § 3 Su xdc dinh va dang chinh tac cia mot phép doi hinh phang 3.1 Trong mat phing dinh huingP cho hai doan thing bing nhau AB va A’B’. j 1g he thiie vé do dai va vé ‘A’A;B,B! va M,M! dé 90 M'lhanh cilia M trong phépddi hinh thuin hoac trong phép dai hin nghich (phan doi hinh) bign A thinh A’, B thanh B' (va eo nhign 1M thanh M’), 3.2 Ching minh dinb ly sau day (neu lén tinh chat dae trung cla phép di hinh thuan trong mat phang). . a . Dinh ly. Phép doi hinh phang bign mot vécto AB thanh mot véeta A’B! cd ciung d9 di va trong moi phép doi hinh thugn, mot vécto da cho bat KY 190 ‘yGi vécto anh ciia né mot géc khong déi (AB, A'B') = y khong déi, goi 18 géc dbi hinh, Tir dé suy ra ring phép di hinh (thugn) 18 mot phép quay hoge mot phép tinh tign tuy theo géc dai hinh cia né khac khong hay bing khong. Va dai véi phép quay xung quanh m6t diém thi g6c ddi hinh chin 1 géc quay. 3.3 Chimg minh cdc ménh dé sau a) Mot phép dang cu trong mat phang néu da cé hai diém bat dong pl 1 P va Q thi moi digm khéc trén dutmg thang (PQ) ciing déu sm bat dong va phép dang cut dé Ia phép doi ximg truc va tryc doi xiig 1a duimg thing (PQ). b) Mot phép dang cy trong mat phang c6 mét diém bat dong O duy nhat 1a mot phép quay xung quanh tam O mét géc y (sai khéc 2k) ndo 6. 3.4 Hay sit dung két qui néu trong bai tip 2.7 (thuge myc 2.5 ea § 2) ve t clia ba phép d6i xit dé bé sung mot I y § 1, trong dé khong han ché diém M phai thudc mién [AABC], ma M 1a mot diém bat ky cita mat phing cia tam gidée ABC. 3.5 Trone mt png cho ba dung thing pin biét A, 2 vi As, KY hia ™ 1 phép doi xting - tryc D(Ay),k = 1, trong d6 f =D, 0D) 0'D;. Tim digu kign Ag néi trén dé g Ad. 3.6 Ky higu fy, fo va fy lin luot a cdc phép d6i xting true D(AD), ‘D(CF) theo thit tu qua cdc dutmg phan giée AD, BE va CF gic ABC. Xét tich cia phép bign hinh f = fs 0 foo fisg = fa fs fon vah = fi 0 fa fs. Ching minh ring a) fg vi h déu la nhiig phép doi ximg truc, xing, b) f= DUB’ trdn noi tigp tam gide ABC; A’, (Z) wen cae canh BC,CA va AB. § 4 Van dung phép doi bai todn hinh hoc phang 4.1 Ching minh ring trong s6 nhiing hinh binh hanh thi duy nbat chi ¢6 hinh thoi ngoai tigp duge mot dudng ton. 4.2 Tren doan vuong géc chung AB cia hai dudg thing song song @ va b (A va B tren b) ta ly hai diém M va N sao cho AM = NB. Trena hin tir M dud luGi mot gée lay diém P va wen b lay digm Q sao cho doan PQ du mot géc vung. Chimg minh ring PQ cing duge nhin tir vuong. ch khoi phue mot hinh ng giée ABCDE di bi xo¥ di nhung chicdn dé lai cic trung digm O;,02,0s,04, va Os ci s AB, BC,CD,DE va EA cha né, 4.4 Trong mat phang cho ba dutmg thing 'y va 2'z déng quy & mot diém O va mot diém P nim ngoai ba duimg thang 46, Chimg minh ring c6 mét tam gide duy nhat nhin x,y,z lam ba duémg trung tryc cita cdc canh va diém P da cho nim trén mot canh ndo dé ciia nd. 4.5 Hinh thang ABCD cé hai day AB = a,CD ‘éc canh ben BC = ¢,DA = d. Cie tia phan giée trong cde géc ZA va ZD cit nhau 6 M va cdc tia phan gic trong cic g6e ZB va ZC cit nhau 6 N. Ching minh MN || AB||CD va tinh khoing céch MN theo a,b,c va d. 4.6 Chitng minh dinh ly Pappus): Lay cde canh AB va AC cita mot tam gi a) Ching minh ring binh binh hinh BCMN dung ra phia ngoai DABC 6 céic eanh ben song song, cling hung va bing PA c6 dien tich bing tng dign tich hai hinh binh hinh vita dumg. b) Tirdinh ly Pap Pugt hay suy ra dinh ly Pitago (Pythagore). 4.7 Mot diém Af nim trong tam giée ABC, chuyén déng song song véi canh BC cho aén khi chuyén dong song song voi AB cho dén khi cit canh BC, rdi lai chuyén dong song song v6i C’A cho dén khi cat AB, wv... Chiing minh rang sau mot s6 bude, quy dao chuyén dong cita diém M s& duge khép kin, 4.8 Gid sit. Ki mot diém bat ky thude mat phing cia tam giée ABC, lay diém M, d6i ximg v6i M qua (BC), Mz d6i ximg v6i My qua (CA) va My doi xiing voi Mz qua (AB). Sau d6, lai tir diém Mg (déng vai tro nhu diém M)

You might also like